Законы ньютона формулировка и формулы: формулы и определения простыми словами первого, второго и третьего законов

Содержание

формулы и определения простыми словами первого, второго и третьего законов

Три закона Ньютона лежат в основе классической физики, хотя за прошедшие годы стало понятно, что они — лишь частный случай теории относительности. В нашей статье разбираем формулы и определения законов Ньютона простыми словами

Борис Михеев

Автор КП

Андрей Найденов

Преподаватель математики и физики онлайн-школы TutorOnline

Вплоть до XVII века мировая наука жила в условиях почти религиозной веры в постулаты, заданные великим философом Аристотелем. Покушение на них воспринималось как ересь и безжалостно наказывалось. Доходило даже до инквизиции. В этих условиях деятельность Галилея, Декарта, Ньютона была не только научным, но и человеческим подвигом. Их открытия сегодня могут быть даже переформулированы, не теряя своего смысла и значения.

История открытия законов Ньютона

Про то, как Ньютон открыл закон всемирного тяготения, знают практически все. Это та самая история про яблоко, которое упало ему на голову. На самом деле, яблоко на голову Ньютона не падало, но все это происходило в осеннем яблоневом саду, где яблоки действительно падали.

А вот как были сформулированы три знаменитых закона Ньютона, ставшие фундаментом классической механики, знают далеко не все. Впервые формулировки этих законов появились в книге Ньютона «Математические начала натуральной философии» (1687 год). Название этого труда достаточно известно именно потому, что в них впервые Ньютон дал определения всех трех законов.

Исаак Ньютон – математик, физик, астроном, механик. Фото: wikipedia.org

Но перед тем, как формулировки этих законов были напечатаны, много чего произошло. Начиная с Древней Греции, многие мыслители пытались облечь в слова фундаментальные законы движения. Потребовалось несколько веков, чтобы сложились предпосылки для этого. Ближе всего к этому подошел Галилей. Но и ему помешали господствующие в научном сообществе иллюзии. Все были безоговорочно уверены, что небесные тела движутся строго по круговым орбитам, потому что это творение Бога, и это творение должно быть совершенно и безупречно. Пошатнуть эти иллюзии удалось Кеплеру. Но и он в своих размышлениях пошел не туда.

В ТЕМУ

Гениальность Ньютона заключается в том, что, изучая труды своих великих предшественников, он смог разглядеть неочевидные вещи, которые даже нам кажутся парадоксальными. Именно Ньютон выдвинул революционную идею, что если на тело не действуют никакие силы, то тело может двигаться прямолинейно и равномерно. В условиях Земли это невозможно, так как действует сила земного тяготения. А вот вне Земли — это обычное дело.

Долгие годы размышлений, черновых набросков, сомнений, которые он выражал в письмах своим коллегам, завершились блестящими формулировками всех трех законов. И эти законы по праву носят имя Ньютона. О каждом из этих законов можно написать отдельную статью — настолько велико и многогранно их значение.

Первый закон Ньютона

Первый закон Ньютона еще называют закон инерции. Фактически он был открыт Галилеем, но именно Исаак Ньютон дал точную его формулировку и включил в число основных законов механики.

Определение

Существуют такие системы отсчёта, называемые инерциальными, относительно которых материальная точка при отсутствии внешних воздействий сохраняет величину и направление своей скорости неограниченно долго. Формулы первый закон Ньютона не имеет.

Второй закон Ньютона

Действие второго закона Ньютона мы можем часто наблюдать в жизни. Возьмём теннисную ракетку и мяч. Если ударить ракеткой по мячу, то мяч приобретёт ускорение равное отношению равнодействующей всех сил к массе.

Определение

В инерциальных системах отсчёта ускорение, приобретаемое материальной точкой, прямо пропорционально вызывающей его силе, совпадает с ней по направлению и обратно пропорционально массе материальной точки.

Формула

Третий закон Ньютона

Третий закон Ньютона объясняет, как, например, двигаются утки. Они находятся во взаимодействии с водой, отталкивая ее назад лапками, а сами благодаря ответному действию двигаются вперед.

Определение

Материальные точки взаимодействуют друг с другом силами, имеющими одинаковую природу, направленными вдоль прямой, соединяющей эти точки, равными по модулю и противоположными по направлению.

Формула

Популярные вопросы и ответы

Отвечает Андрей Найденов, преподаватель физики и математики в онлайн-школе TutorOnline

Сколько законов Ньютона изучают в школе?

— В школе изучают четыре закона Ньютона. Первые три являются фундаментом классической динамики. С их помощью можно описать любое движение тела, которое движется со скоростью, значительно меньшей скорости света.

Особняком стоит закон всемирного тяготения. Он лежит в основе классической теории гравитации. Этот закон перекликается со вторым законом, касающемся соотношения между ускорением тела, его массой и действующей на него силой. Но все же это разные законы. Так как второй закон Ньютона более универсален, чем закон всемирного тяготения.

Как и где выполняются законы Ньютона в жизни?

— Эти законы настолько фундаментальны, что увидеть их можно практически всюду. Пуля из ружья летит туда, куда толкают ее пороховые газы. Ракета летит туда, куда толкают ее продукты горения ракетного топлива. Мяч летит туда, куда пинает его футболист. Это примеры действия первого закона Ньютона.

Второй закон связан с ускорением, которое приобретает тело под действием силы, действующей на это тело. Например, болид «Формулы 1» разгоняется намного быстрее, чем простой серийный легковой автомобиль. На них действуют разные по величине силы тяги их двигателей. Мощность двигателя в спортивной машине на порядок выше, чем у обыкновенной.

Третий закон Ньютона устанавливает закон взаимодействия тел. Сила действия равна силе противодействия. Например, если чашка кофе стоит на столе и не проваливается сквозь него на пол, значит, стол оказывает достаточное противодействие силе тяжести, действующей на чашку. В результате эти две силы уравновешивают друг друга, и чашка стоит на столе без какого-либо движения.

Утка находится во взаимодействии с водой, отталкивая ее назад лапками, а сама благодаря ответному действию движется вперед. Фото pixabay.com

Как Ньютон изменил третий закон Кеплера?

— Законы Кеплера — это три соотношения, которые выведены Кеплером на основе астрономических наблюдений. Эти законы описывают идеализированную планету, движущуюся по круговой орбите. Опираясь на третий закон Кеплера, Ньютон сформулировал закон всемирного тяготения. При этом третий закон Кеплера в рамках теории всемирного тяготения Ньютона является решением частной задачи, которая касается двух тел. Другими словами, третий закон Кеплера стал частью более общей теории Ньютона.

Фото на обложке: pixabay.com

Законы Ньютона. Формулы и кратко объяснение

Классическая механика Ньютона наглядно объясняет закономерности нашего мира при скоростях, далеких от скорости света. В ее основе лежат три закона, которые английский ученый Исаак Ньютон впервые сформулировал в 1687 г. в своей книге «Математические начала натуральной философии». Надо сказать, что сегодня они формулируются несколько по-другому — более точно.

Кембриджский университет, где учился и работал И. Ньютон

Формулировка первого закона была следующей: всякое тело продолжает удерживаться в своем состоянии покоя или равномерного и прямолинейного движения, пока и поскольку оно не понуждается приложенными силами изменять это состояние. Сегодня закон излагают немного не так, потому что Ньютон опирался на неподвижную систему отсчета, то есть на абсолютное пространство-время, с чем современная физика не согласна. Кроме того, понятие «тело» заменено на понятие «материальная точка», потому что тело конечных размеров в отсутствие внешних сил способно также вращаться. Таким образом, первый закон утверждает, что, если уже движущееся тело не трогать, оно будет по инерции продолжать двигаться по прямой. Инерция — это такое свойство тела, при котором скорость его движения остается неизменной и по величине, и по направлению, когда на него не действуют никакие силы. Чтобы изменить скорость движения тела или заставить неподвижное тело двигаться, на него нужно воздействовать с определенной силой. Разумеется, одинаковые силы воздействуют на различные тела по-разному. Иными словами, у тел имеется различная инертность, то есть свойство сопротивляться изменению скорости. Поэтому первый закон Ньютона называют также законом инерции.

Современная формулировка звучит следующим образом: существуют такие системы отсчета, называемые инерциальными, относительно которых материальные точки, когда на них не действуют никакие силы (или действуют силы взаимно уравновешенные), находятся в состоянии покоя или равномерного прямолинейного движения.

Схема, иллюстрирующая действие первого закона Ньютона

Итак, с помощью первого закона Ньютона определяется, находится ли тело под воздействием внешних сил.

Второй закон Ньютона в классической формулировке выглядит так: изменение количества движения пропорционально приложенной движущей силе и происходит по направлению той прямой, по которой эта сила действует.

Современная формулировка звучит так: в инерциальной системе отсчета ускорение, которое получает материальная точка с постоянной массой, прямо пропорционально равнодействующей всех приложенных к ней сил и обратно пропорционально ее массе.

Схема, иллюстрирующая действие второго закона Ньютона

Таким образом, второй закон показывает, что происходит с телом (материальной точкой) под воздействием внешних сил. Кроме того, он вводит массу как меру проявления инертности и связывает ее с ускорением. Чем больше приложенная к телу сила (точнее, сумма сил), тем большее ускорение оно приобретает, а чем массивнее это тело, тем меньше ускорение. В виде формулы второй закон записывается так:

F=ma

где F — сила, m — масса, a — ускорение. Отсюда видно, что ускорение обратно пропорционально массе, то есть чем больше масса, тем меньше ускорение при приложении определенной силы.

И если знать направление и интенсивность всех действующих на точку (тело) сил, а также ее координаты и скорость, то можно предсказать будущее состояние этой точки. Такая концепция существовала в науке более 200 лет, пока не появилась квантовая механика.

Третий закон в классической формулировке выглядит следующим образом: действию всегда есть равное и противоположное противодействие, иначе — взаимодействия двух тел друг на друга между собою равны и направлены в противоположные стороны.

Сегодня он звучит так: материальные точки взаимодействуют друг с другом силами, имеющими одинаковую природу, направленными вдоль прямой, соединяющей эти точки, равными по модулю и противоположными по направлению.

Иными словами, согласно этому закону, любая сила является результатом взаимодействия других сил. Однако они могут значительно отличаться по величине. Когда с дерева падает яблоко, не только Земля притягивает его, но и оно притягивает Землю, и центр планеты смещается вверх, навстречу яблоку. Но на ничтожно малое расстояние — меньше диаметра атомного ядра. Ведь масса Земли, а значит, и ее инертность, несопоставимо больше массы яблока. И здесь уже работает второй закон Ньютона.

Схема, иллюстрирующая действие третьего закона Ньютона

Вообще тела при движении нередко наглядно демонстрируют действие всех трех законов. Например, при запуске ракеты на нее действует сила реактивной тяги, и ракета поднимается (третий закон Ньютона), причем с ускорением (второй закон Ньютона), а выйдя на орбиту, начинает двигаться по ней согласно первому закону Ньютона.

Можно заметить, что в современных формулировках используется термин «инерциальная система отсчета». Это значит, что в неинерциальных системах отсчета законы Ньютона должны выполняться с поправками на силу инерции. Неинерциальные системы — такие, которые движутся с ускорением относительно инерциальных.

Законы Ньютона положили начало классической физике, именно их используют, чтобы строить дома и конструировать машины, запускать космические корабли. Однако с появлением квантовой механики в физике появились новые законы, не столь наглядные, как законы классической механики. Тем не менее законы Ньютона справедливы в очень широкой области и их невозможно отменить.

Падало ли яблоко на голову Ньютона

Исаак Ньютон (1643—1727) — великий английский ученый, физик и математик, создатель классической механики и научной картины мира, которая доминировала в науке более 200 лет

Ньютон также открыл закон всемирного тяготения. Существует исторический анекдот, что ученый сделал это после того, как ему на голову упало яблоко. Рассказывают также, что все свои открытия Ньютон совершил во время так называемых чумных каникул, в 1665—1666 гг., когда в Англии бушевала чума и он вынужден был уехать из Кембриджа, где учился и работал, в деревню. На самом же деле открытия Ньютона были результатом работы многих лет.

Физика, проникнутая математикой

Механиков и математиков готовят на одном и том же факультете

Законы Ньютона являются основными законами механики, более того, с них началась математизация физики. Механика настолько проникнута математикой, что специалистов по ней готовят на механико-математических (или математико-механических) факультетах, а не на физических.

Поделиться ссылкой

Законы Ньютона: кратко и понятно о формулах и формулировках на конкретных примерах

В школьном курсе физики изучаются три закона Ньютона, являющиеся основой классической механики. Сегодня с ними знаком каждый школьник, но во времена великого ученого подобные открытия считались революционными. Законы Ньютона, кратко и понятно будут описаны ниже, они помогают не только понять основу механики и взаимодействия объектов, но и помогают записать данные в качестве уравнения.

Содержание

Впервые три закона Иссак Ньютон описал в труде «Математические начала натуральной философии» (1867 год), в котором были подробно изложены не только собственные выводы ученого, но все знания по этой теме открытые другими философами и математиками. Таким образом, труд стал фундаментальным в истории механики, а позднее и физики. В нем рассмотрены перемещение и взаимодействие массивных тел.

Интересно знать! Исаак Ньютон был не только талантливым физиком, математиком и астрономом, но и считался гением в механике. Занимал должность президента королевского общества Лондона.

Каждое утверждение освещает одну из сфер взаимодействия и перемещения предметов в природе, правда обращение к ним было несколько упразднено Ньютоном, и они были приняты как точки без определенного размера (математические).

Именно это упрощение позволило проигнорировать естественные физические явления: воздушное сопротивление, трение, температуру или другие физические показатели объекта.

Полученные данные могли быть описаны только по времени, массе или длине. Именно из-за этого формулировки Ньютона обеспечивают лишь подходящие, но приближенные значения, которые нельзя использовать для описания точной реакции крупных или изменяемых по форме объектов.

Перемещение массивных предметов, которые участвуют в определениях, принято исчислять в инерциальной системе отсчета, представленной в виде системы координат из трех измерений, и при этом она не увеличивает свою скорость и не оборачивается вокруг своей оси.

Ее часто называют системой отсчета Ньютона, но при этом ученый никогда не создавал и не использовал подобной системы, а использовал нерациональную. Именно в этой системе тела могут двигаться так, как описывает это Ньютон.

Первый закон

Называется законом инерции. Не существует его практической формулы, зато есть несколько формулировок. В учебниках по физике предлагается следующая формулировка первого закона Ньютона: есть инерциальные системы отсчета, в отношении которых объект, если он свободен от воздействия любых сил (или же они моментально компенсируется), находиться в полном покое или же двигается по прямой и с одинаковой скоростью. Что означает данное определение и как его понять?

Простыми словами первый закон Ньютона объясняется так: любое тело, если его не трогать и никоим образом не воздействовать на него, будет оставаться постоянно в состоянии покоя, то есть бесконечно стоять на месте. То же самое происходит и при его движении: оно будет равномерно двигаться по заданной траектории бесконечно, пока на него не воздействует что-либо.

Подобное утверждение озвучивал Галилео Галилей, но не смог уточнить и точно описать это явление. В этой формулировке важно правильно понять, что такое инерциальные системы отсчета. Если сказать совсем простыми словами, то это система, в которой выполняется действие данного определения.

Это интересно! Изучаем термины: энтропия – что же это такое простыми словами

Первый закон Ньютона

В мире можно увидеть огромное множество подобных систем, если понаблюдать за движением:

  • поезда на заданном участке с одинаковой скоростью,
  • Луны вокруг Земли,
  • колеса обозрения в парке.

Это интересно! Как правильно перевести МПА атмосферы

В качестве примера рассмотрим некоего парашютиста, который уже раскрыл парашют и движется прямолинейно и при этом равномерно по отношению к поверхности Земли. Движение человека не прекратиться до тех пор, пока земное притяжение будет компенсироваться движением и сопротивлением воздуха. Как только это сопротивление уменьшится, то притяжение увеличится, что приведет к изменению скорости парашютиста – его движение станет прямолинейным и равноускоренным.

Именно в отношении этой формулировки существует яблочная легенда: Исаак отдыхал в саду под яблоней и размышлял о физических явлениях, когда с дерева сорвалось спелое яблоко и упало в траву. Именно ровное падение заставило ученого изучить этот вопрос и выдать в итоге научное объяснение движению предмета в некой системе отсчета.

Интересно знать! Помимо трех явлений в механике, Исаак Ньютон также объяснил движение Луны как спутника Земли, создал корпускулярную теорию света и разложил радугу на 7 цветов.

Второй закон

Данное научное обоснование касается не просто движения предметов в пространстве, а взаимодействия их с другими объектами и результатов этого процесса.

Закон гласит: увеличение скорости объекта с некоторой постоянной массой в инерциальной системе отсчета прямо пропорционально силе воздействия и обратно пропорционально постоянной массе движущегося предмета.

Проще говоря, если существует некое движущиеся тело, масса которого не изменяется, и на него вдруг начнет воздействовать посторонняя сила, то оно начнет ускоряться. А вот скорость ускорения будет прямо зависеть от воздействия и обратно пропорционально зависеть от массы движущегося предмета.

Для примера можно рассмотреть снеговой шар, который катиться с горы. Если шар толкать по ходу движения, то ускорения шара будет зависеть от мощности воздействия: чем она больше, тем больше ускорение. Но, чем больше масса данного шара, тем меньше будет ускорение. Данное явление описывается формулой, в которой учитывается ускорение, или «a», равнодействующая масса всех воздействующих сил, или «F», а также масса самого предмета, или «m»:

а = F/m

Следует уточнить, что данная формула может существовать только в том случае, если равнодействующая всех сил не меньше и не равна нулю. Применяется закон только относительно тел, которые двигаются со скоростью меньше световой.

Это интересно! Квантовые постулаты Нильса Бора: кратко об основных положениях

Полезное видео: первый и второй законы Ньютона

Третий закон

Многие слышали выражение: «На каждое действие есть свое противодействие». Его часто используют не только в общеобразовательных целях, но и воспитательных, объясняя, что на каждую силу найдется большая.

Эта формулировка пошла от очередного научного утверждения Исаака Ньютона, а точнее его третьего закона, который объясняет взаимодействие различных сил в природе относительно какого-либо тела.

Третий закон Ньютона определение имеет такое: предметы оказывают воздействие друг на друга с силами одинаковой природы (соединяющей массы предметов и направлены вдоль прямой), которые равны по своим модулям и при этом направлены в разные стороны. Данная формулировка звучит достаточно сложно, но простыми словами объяснить закон легко: каждая сила имеет свое противодействие или равную силу, направленную в обратную сторону.

Гораздо проще будет понять смысл закона, если в качестве примера взять пушку, из которой стреляют ядрами. Пушка воздействует на снаряд с той же силой, с которой снаряд воздействует на пушку. Подтверждением этого будет небольшое движение пушки назад во время выстрела, что подтвердит воздействие ядра на орудие. Если взять как пример тоже самое яблоко, которое падает на землю, то станет понятно, что яблоко и земля воздействуют друг на друга с равной силой.

Это интересно! В чем заключается принцип теории Гюйгенса Френеля

Закон имеет также математическое определение, в котором используется сила первого тела (F1) и второго (F2):

F1 = -F2

Знак минуса сообщает о том, что векторы сил двух разных тел направлены в противоположные стороны. При этом важно помнить, что данные силы не компенсируют друг друга, поскольку направлены относительно двух тел, а не одного.

Полезное видео: 3 закона Ньютона на примере велосипеда

Вывод

Данные законы Ньютона кратко и четко необходимо знать каждому взрослому человеку, поскольку они являются основой механики и действуют в повседневной жизни, несмотря на то, что не при всех условиях данные закономерности соблюдаются. Они стали аксиомами в классической механике, и на основе их были созданы уравнения движения и энергии (сохранение импульса и сохранение механической энергии).

Кратко и понятно о первом, втором и третьем законах Ньютона: формулировки, примеры и формулы

Главная » Наука

Три закона Ньютона — это основа классической механики. В 1867 году Ньютон опубликовал работу под названием «Математические начала натуральной философии». Там были все знания, накопленные до него другими учёными, а также новые, открытые самим Ньютоном. Его считают одним из самых первых основоположником современной физики. Благодаря систематизированным знаниям, которые были описаны в вышеуказанном труде, он открыл множество законов механики, Закон всемирного тяготения и многое другое.

Оглавление:

  • Кратко о законах Ньютона
  • Первый закон Ньютона
  • Второй закон Ньютона
  • Третий закон Ньютона

Содержание

Первый закон Ньютона

  1. Формулировка. В наше время встречаются несколько формулировок, вот одна из самых современных: «Существуют такие инерциальные системы отсчёта, относительно которых тело, если на него не действуют другие силы (либо действие других сил компенсируется), находится в покое либо движется равномерно и прямолинейно». Этот закон иногда называют Законом инерции.
  2. Трактовка. Если описать это утверждение простыми словами, то можно увидеть, что всё достаточно просто: если какое-то тело находится в покое относительно чего-либо, то оно и будет оставаться в покое до тех пор, пока на него не подействует какой-либо предмет. То же самое, если тело движется равномерно прямолинейно, то оно будет продолжать так двигаться, пока на него не подействует какая-либо сила. До Ньютона его открыл Галилео Галилей, но он не совсем точно его описал. Теперь осталось только разобраться, что такое инерциальные системы отсчёта. Проще говоря, это такая система, для которой выполняется Первый закон Ньютона.
  3. Пример действия. Представьте себе парашютиста, который движется прямолинейно равномерно к Земле. Это будет продолжаться до тех пор, пока притяжение к поверхности Земли будет компенсироваться сопротивлением воздуха. Если же сопротивление станет меньше либо больше, то тогда на тело начнёт действовать сила притяжения, и оно станет двигаться прямолинейно равноускоренно.
  4. История открытия. Существует легенда об открытии этого утверждения. Когда-то Ньютон сидел под деревом, и рядом с ним упало яблоко. Это подтолкнуло его на размышления о том, почему яблоко упало перпендикулярно земле, каковы были причины данного явления. По крайней мере, так описывал этот эпизод знаменитый биограф Уильям Стьюкли.
  5. Формулы у него нет.

Это интересно: система отсчета в физике определение и ее виды.

Второй закон Ньютона

Он описывает поведение тела при действии на него других объектов. Что с ним происходит, как он начинает двигаться и прочее.

  1. Формулировка. «В инерциальных системах отсчёта ускорение тела с постоянной массой прямо пропорционально равнодействующей всех сил и обратно пропорционально его массе».
  2. Формула. Математическое описание этого утверждения такое: а = F/m, где a — это ускорение, F — равнодействующая всех сил, приложенных к телу, m — масса тела.
  3. Трактовка. Из формулы мы видим, что ускорение тела зависит от силы, приложенной к этому телу, и массы. А также можно увидеть, что чем больше равнодействующая всех сил, то тем больше ускорение, и чем больше масса тела, тем ускорение меньше. Говоря простым языком, если равнодействующая всех сил не равна нулю и не меньше нуля, то выполняется данное утверждение. Можно сказать ещё проще, если на тело действует сила, то оно приобретает ускорение.
  4. Пример действия. Возьмём бейсбольную биту и мяч. Если ударить битой по мячу, и удар будет сильнее действия всех других сил, то мяч приобретёт ускорение равное отношению равнодействующей всех сил к массе.

Это интересно: формула всемирного тяготения определение закона.

Третий закон Ньютона

  1. Формулировка. «Тела взаимодействуют друг на друга с силами одинаковой природы, направленными вдоль прямой, которая соединяет центры масс этих тел, а силы равны по модулю и разнонаправленны».
  2. Трактовка. Это значит, что на каждое действие есть своё противодействие.
  3. Пример действия. Более понятно это можно рассмотреть на таком примере: представьте пушку, из которой стреляют ядром. Ядро будет действовать на пушку с той же силой, с какой пушка вытолкала ядро. Поэтому при выстреле пушка откатится чуть-чуть назад, это происходит из-за того, что размеры пушки и ядра разные. Примерно то же самое происходит и при падении яблока на землю. Земля действует на яблоко с некой силой и яблоко тоже действует на Землю. Только из-за того, что масса Земли в миллионы раз больше яблока этого действия не видно. Еще один пример действия Третьего закона для закрепления усвоенного. Возьмём довольно сложный пример: притяжение планет. Луна вертится вокруг Земли благодаря тому, что она притягивается к Земле, но по Третьему закону Ньютона Луна тоже притягивает Землю к себе. Однако, из-за того, что их массы разные, Луна не может притянуть Землю, но у неё получается вызвать отливы и приливы в морях и океанах.
  4. Формула. Математически это утверждение можно записать так: F1 = -F2, где F1 — это сила, с которой первое тело действует на второе, а F2 — сила, с которой второе тело действует на первое.

Первый закон Ньютона – формула и определение кратко и понятно об инерциальной системе отсчета

4.2

Средняя оценка: 4.2

Всего получено оценок: 469.

4.2

Средняя оценка: 4.2

Всего получено оценок: 469.

Мир полон движения: движутся звезды и планеты и на нашей планете мы также видим движение всюду – течет вода в реках, ветер гонит облака и качает деревья, по дорогам едут автомобили, по рельсам – поезда, в воздухе летят самолеты. Наукой доказано движение невидимых глазом частиц – молекул, атомов. Движение является основным свойством материи и подчиняется законам Ньютона.

Закон инерции, или Первый закон Ньютона

Механическое движение характеризуется скоростью. И вот другое основное положение, которое утверждает, что движущееся тело не может само по себе изменить свою скорость. Если на движущееся тело не действуют никакие другие тела, то тело не может ни ускориться, ни замедлиться, ни изменить направление своего движения, оно будет двигаться с какой-то определенной по модулю и направлению скоростью. Только воздействие тел извне может изменить эту скорость.

Свойство тел сохранять модуль и направление своей скорости называется инерцией

Первым явление инерции объяснил Галилей. Ньютон же сформулировал «закон инерции». Формулировка его звучит следующим образом: всякое тело сохраняет состояние покоя или равномерного и прямолинейного движения, пока действия со стороны других тел не изменят этого состояния.

Рис. 1. Портрет Ньютона.

Ни один предмет сам собой не придет в движение. Стоящий в комнате стол никогда сам собой не начнет двигаться по комнате. Движущееся тело не может само собой остановиться.

Когда водитель трамвая резко тормозит, то находящиеся в вагоне пассажиры помимо воли наклоняются вперед, продолжая свое движение по инерции.

Резко трогающийся с места поезд метрополитена заставляет пассажиров отступать или откидываться назад.

А на крутом повороте дороге можно вылететь из санок в сугроб.

Примеров инерции существует огромное множество. Инерционность – неотъемлемое свойство движущейся материи.

Что же может произойти в мире, если бы мгновенно исчезло свойство тел, которое мы называем инерцией. Луна упала бы на Землю, а планеты – на Солнце. Движение тела осуществлялось бы только под действием силы и прекращалось с исчезновением последней. Даже больше: исчезновение инерции означало бы исчезновение самого движения. Таким образом, инерция есть не что иное, как выражение единства материи и движения.

Рис. 2. Солнечная система.

И в природе, и в технике нет тел, на которые не действовали бы другие тела. Например, на тело, находящееся на столе, действует опора и Земля. Тело находится в покое, потому что действия опоры и Земли уравновешивают друг друга. Опускаясь на парашюте, парашютист движется равномерно и прямолинейно (V=const), несмотря на то, что на него действует Земля и воздух. Ракета вдали от звезд будет также двигаться равномерно и прямолинейно, так как на нее не будут действовать другие тела.

Движение одних тел под действием других тел подчиняется законам динамики

Галилей, исходя из многочисленных наблюдений пришел к выводу, что если действия нет или все действия скомпенсированы (равнодействующая всех сил равна 0; R=0), то тело покоится или движется равномерно и прямолинейно (V=const; a=0).

Но движение тела необходимо рассматривать относительно других тел, иначе невозможно будет определить положение тела в пространстве. Значит, говоря о явлении инерции, необходимо указать, относительно чего тело покоится или движется равномерно и прямолинейно.

Поэтому первый закон Ньютона, названный законом инерции, также носит следующее определение:

Существуют системы отсчета, относительно которых поступательно движущееся тело сохраняет свою скорость постоянной, если действие на него других тел скомпенсировано.

Формулы первого закона Ньютона не существует.

Инерциальные системы отсчета

Системы отсчета, которые упоминаются в первом законе Ньютона, называются инерциальными системами отсчета (ИСО).

Какие же системы отсчета можно отнести к инерциальным?

  • те, в которых при R=0; V=const
  • те, которые движутся относительно ИСО равномерно и прямолинейно (например, звезды). На самом деле не существует такой ситуации, при которой на тело не действовали другие тела. Однако, если действие одних тел скомпенсировано, а действие других слишком мало, то принято считать, что в определенном приближении на тело никакие тела не действуют.
Рис. 3. Инерциальные и неинерциальные системы отсчета.

Солнце и Земля не являются инерциальными системами отсчета. Но эффекты, вызванные их неинерцианальностью, незначительны. в ряде случаев ими можно пренебречь, правда не всегда

Первый закон Ньютона выполняется не во всех СО, а только в инерциальных. Во всех ИСО при первоначальных одинаковых условиях механические явления протекают одинаково, то есть подчиняются одинаковым законам. Это утверждение носит название – принцип относительности Галилея.

Все ИСО равноправны:

Никакими механическими опытами, проведенными в пределах данной системы, нельзя установить, находится ли она в состоянии покоя или в состоянии равномерного и прямолинейного движения.

Что мы узнали?

В данной статье кратко и понятно объясняется первый закон Ньютона, инерциальные системы отсчета и их взаимосвязь. Ведь, как известно, первый закон Ньютона действителен только для инерциальных систем отсчета.

Тест по теме

Доска почёта

Чтобы попасть сюда – пройдите тест.

  • Данил Горжий

    7/10

  • Егор Чаплин

    10/10

Оценка доклада

4.2

Средняя оценка: 4.2

Всего получено оценок: 469.


А какая ваша оценка?

Второй закон Ньютона – формула, запись и определение кратко » Kupuk.net

Второй закон Ньютона – фундамент классической механики, который дает понятие о причинах движения. На его основе строится решение многих прикладных задач и задач школьного курса физики. Поэтому важно понимать его суть и уметь применять на практике.

Формулировка второго закона Ньютона

В первоначальной формулировке закона было сказано, что изменение количества движения (то есть импульса) пропорционально силе, заставляющей тело двигаться, и направлено по направлению действия силы.

Математически это можно записать так:

$$vec{F} = {dvec{p} over vec{dt}}$$

Буква d от delta – изменение, разность. Называется дифференциалом, а частное двух дифференциалов, в котором одно значение – переменная (время t), другое – функция (импульс p) – производная. То есть, это скорость изменения функции.

Рис. 1. Геометрический смысл производной.

Но: $vec{p} = {mvec{v}}$

Тогда уравнение (2) можно переписать в следующем виде:

$$vec{F} = m{dvec{v} over vec{dt}}$$

Второй множитель – это определение ускорения. Ускорение есть быстрота изменения скорости движения.

Тогда запись второго закона Ньютона примет привычный вид:

$$vec{F} = mvec{а}$$

Или, разделив обе части на m:

$vec{a} = {vec{F} over {m}}$ – ускорение, с которым движется тело, прямо пропорционально приложенной к нему силе и обратно пропорционально его массе. Чем больше масса, тем меньше ускорение, чем больше сила, тем больше ускорение. Данный закон также называют основным законом классической механики.

Под F здесь понимают геометрическую сумму всех действующих на тело (то есть внешних) сил. Иными словами, их равнодействующую.

Геометрическая сумма – это сумма векторных величин. Сложение векторов выполняется по правилу треугольника или параллелограмма. Поэтому для расчета равнодействующей сил необходимы значение модулей сил и углы между силами.

Рис. 2. Нахождение равнодействующей сил.

Второй закон справедлив для инерциальных и неинерциальных систем отсчета, для материальных точек и произвольных тел, так как любое тело можно представить множеством материальных точек (местом приложения силы будет центр масс тела.

Для расширения второго закона Ньютона на случай неинерциальных систем отсчета вводят понятие инерциальных сил, таких, например, как центробежная сила или сила Кориолиса. Математическая формулировка будет выглядеть так: $ mvec{а}={vec{F} + vec{F_и}}$. Индексом “и” обозначены инерциальные силы.

Применение основного закона классической динамики

Рассмотрим тело, на которое действует несколько сил. Допустим, это машина, застрявшая в грязи. Ее пытается вытянуть вторая машина, зацепив ее тросом. Таким образом, первая машина стремится изменить свое количество движения под действием силы натяжения троса, силы тяги и сил трения. Так будет выглядеть формула второго закона Ньютона для первой машины:

$$mvec{а}={vec{T} + vec{F_{тр}} + vec{F_т}}$$

Или в скалярной форме:

$$mа={T – F_{тр} + F_т}$$

Теперь же предположим, что равнодействующая сил обращает в нуль (силы трения, силы тяги и силы натяжения нити). Тогда машина будет покоиться или двигаться равномерно и прямолинейно. Это утверждение можно расширить на случай системы тел: если при взаимодействии друг с другом частей системы внутренние силы суммарно равны нулю, а внешние силы скомпенсированы или не действуют, то система тел покоится или движется равномерно.

Рис. 3. Исаак Ньютон. 2 – F_1F_2}$$ – окончательный ответ.

  • Через блок перекинута веревка. На одном ее конце закреплен груз. На другом конце висит альпинист той же массы, что и груз. Что будет с системой, если альпинист начнет карабкаться вверх? Массой веревки и трением в блоке пренебречь.

Решение второй задачи

Запишем второй закон Ньютона для альпиниста и для груза.

$$mа_1={T_1 – mg}$$

$$mа_2={T_2 – mg}$$

Но $T_1={T_2}$

Поэтому:

$${mа_1} = {mа_2}$$

Сократив массы, получим, что их ускорения одинаковы. Альпинист и груз будут подниматься с одинаковыми ускорениями и достигнут блока одновременно. Если бы масса альпиниста была меньше массы груза, то груз поднимался бы быстрее.

Что мы узнали?

В ходе урока был сформулирован второй закон Ньютона в его современной форме, выяснена его сущность и рассмотрены примеры его использования в реальных ситуациях. В завершение урока были кратко разобраны две простые задачи на нахождение равнодействующей сил и на применение второго закона Ньютона.

2.3 Законы Ньютона | Законы Ньютона

Предыдущий

2.2 Сила

Следующий

2.4 Силы между массами

2.3 Законы Ньютона (ЭСБКР)

В этом разделе мы рассмотрим влияние сил на объекты и то, как мы можем заставить их двигаться. Это будет свяжите воедино то, что вы узнали о движении, и то, что вы узнали о силах.

Первый закон Ньютона (ESBKS)

Сэр Исаак Ньютон был ученым, жившим в Англии (1642-1727), который интересовался движением объектов в различных условиях. Он предположил, что неподвижный объект останется неподвижным до тех пор, пока сила действует на него, и что движущийся объект будет продолжать двигаться, пока сила не замедлит его, не ускорит его вверх или меняет направление движения. Отсюда он сформулировал то, что известно как первый закон Ньютона. движения:

Первый закон движения Ньютона

Объект продолжает находиться в состоянии покоя или равномерного движения (движения с постоянной скоростью), если только на него действует неуравновешенная (чистая или равнодействующая) сила.

Это свойство объекта продолжать свое текущее состояние движения, если на него не действует результирующая сила, называется инерция .

Рассмотрим следующие ситуации:

Фигуристка отталкивается от края катка и катится по льду. Она будет продолжать двигаться по прямой линии по льду, если ее что-то не остановит. Предметы тоже похожи что. Если мы ударим футбольным мячом по футбольному полю, то, согласно первому закону Ньютона, футбольный мяч должны двигаться вечно! Однако в реальной жизни этого не происходит. Закон Ньютона неверен? Нет В самом деле. Первый закон Ньютона применим к ситуациям, когда нет никаких внешних сил. Этот значит трения нет. В случае фигуриста трение между коньками и льда очень мало, и она будет продолжать движение довольно далеко. В случае с футболом мяч, сопротивление воздуха (трение между воздухом и мячом) и трение между травой и мячом присутствует, и это замедлит мяч. 9{-1}$}\) по первому закону Ньютона. Если они пристегнуты ремнями безопасности, ремни безопасности остановят их, воздействуя на них силой и так далее. не дать им пострадать.

Видео: 23H8

Ракеты :

Космический корабль запущен в космос. Сила взрывающихся газов проталкивает ракету через воздуха в космос. Как только он оказывается в космосе, двигатели выключаются, и он продолжает движение с постоянная скорость. Если астронавты хотят изменить направление космического корабля, им нужно стрелять двигатель. Это приложит силу к ракете, и она изменит свое направление.

Рабочий пример 8: первый закон Ньютона в действии

Почему пассажиров отбрасывает в сторону, когда машина, в которой они едут, объезжает угол?

Что происходит до поворота автомобиля

Перед поворотом автомобиля пассажиры и автомобиль движутся одновременно скорость. (рисунок А)

Что происходит, когда машине исполняется 9 лет?0027

Водитель поворачивает колеса автомобиля, которые затем воздействуют на автомобиль и автомобиль повороты. Эта сила действует на автомобиль, но не на пассажиров, поэтому (по первому закону Ньютона) пассажиры продолжают движение с той же начальной скоростью. (рисунок Б)

Почему пассажиров отбрасывает в сторону?

Если пассажиры пристегнуты ремнями безопасности, они будут оказывать давление на пассажиров до тех пор, пока скорость пассажиров такая же, как и у автомобиля (рисунок C). Без ремня безопасности пассажир может удариться о борт автомобиля.

Учебник Упражнение 2.4

Если в машине сидит пассажир, а машина поворачивает направо, что происходит с пассажиром? Что будет, если машина повернет налево?

Перед началом поворота автомобиля пассажир и автомобиль движутся одновременно скорость.

Когда автомобиль поворачивает направо, сила действует на автомобиль, но не на пассажиров, поэтому (по Первый закон Ньютона) пассажир продолжает двигаться с той же скоростью. (В то есть машина поворачивает, а пассажир нет).

Конечным результатом этого является то, что пассажира тянет влево, когда машина поворачивает Правильно.

Если бы машина вместо этого повернула налево, пассажира потянуло бы направо.

Гелий менее плотный, чем воздух, которым мы дышим. Обсудите, почему гелиевый шар в машине вождение за угол, кажется, нарушает первый закон Ньютона и движется в сторону внутри поворота не снаружи как у пассажира.

По мере того, как машина поворачивает, весь воздух продолжает двигаться вперед (он действует так же поведение пассажира). Это приводит к тому, что давление воздуха с одной стороны автомобиля увеличение (это будет на стороне, противоположной направлению поворота автомобиля). Этот небольшое увеличение давления воздуха толкает гелиевый шар на другую сторону автомобиль.

Из-за этого кажется, что гелиевый шар не подчиняется первому закону Ньютона.

Второй закон движения Ньютона (ESBKT)

Согласно первому закону Ньютона, вещи «любят продолжать делать то, что они делают». Другими словами, если объект движется, он стремится продолжать движение (по прямой линии и с той же скоростью), и если объект неподвижен, он стремится оставаться неподвижным. Так как же объекты начинают двигаться?

Давайте посмотрим на пример коробки \(\text{10}\) \(\text{kg}\) на грубом столе. Если мы слегка нажмем на коробка, как показано на диаграмме, коробка не будет двигаться. Допустим, мы приложили силу \(\text{100}\) \(\text{N}\), но коробка остается неподвижной. В этот момент сила трения \(\text{100}\) \(\text{N}\) действует на коробку, не давая ей двигаться. Если мы увеличим силу, скажем к \(\text{150}\) \(\text{N}\) и коробка почти начинает двигаться, сила трения равна \(\text{150}\) \(\текст{N}\). Чтобы сдвинуть коробку, нам нужно приложить достаточно сильное усилие, чтобы преодолеть трение, а затем переместить коробку. Поэтому, если мы приложим силу \(\text{200}\) \(\text{N}\), помня, что фрикционное сила из \(\text{150}\) \(\text{N}\) будет использоваться “первый” \(\text{150}\) \(\text{N}\) для преодоления или «отменить» трение, а другой \(\text{50}\) \(\text{N}\) будет использоваться для перемещения (ускорения) блокировать. Чтобы ускорить объект, мы должны иметь результирующую силу, действующую на блок.

Теперь, как вы думаете, что произойдет, если мы нажмем сильнее, скажем, \(\text{300}\) \(\text{N}\)? Или что делать вы думаете, произойдет, если масса блока будет больше, скажем, \(\text{20}\) \(\text{kg}\), или что, если он было меньше? Давайте исследуем, как на движение объекта влияют масса и сила.

Рекомендуемый эксперимент для формальной оценки второго закона Ньютона также включен в эта глава. В этом эксперименте учащиеся будут исследовать взаимосвязь между силой и ускорения (второй закон Ньютона). Вам понадобятся тележки, разные массы, наклонная плоскость, резина ленты, измерительная линейка, тикерная лента, тикерный таймер, миллиметровка.

Второй закон движения Ньютона

Цель

Исследовать связь между ускорением объектов и применением постоянной Равнодействующая сила.

Метод

  1. Постоянная сила \(\text{20}\) \(\text{N}\), действующая под углом \(\text{60}\)\(\text{°}\) к горизонтали, применяется к тележке динамики.

  2. Тикерная лента, прикрепленная к тележке, проходит через бегущий таймер частоты \(\text{20}\) \(\text{Гц}\) при движении тележки по поверхности без трения.

  3. Описанная выше процедура повторяется 4 раза, каждый раз с использованием одной и той же силы, но с различной масса тележки следующая:

    • Случай 1: \(\text{6,25}\) \(\text{кг}\)

    • Случай 2: \(\text{3,57}\) \(\text{кг}\)

    • Случай 3: \(\text{2,27}\) \(\text{кг}\)

    • Случай 4: \(\text{1,67}\) \(\text{кг}\)

  4. Ниже показаны фрагменты четырех полученных бегущих строк. Ленты имеют маркировку буквы A, B, C, D и т. д. A — первая точка, B — вторая точка и т. д. Расстояние между каждой точкой также показано. 9{-1}$}\)) из тележку в точках B и F (не забудьте сначала преобразовать расстояния в m!). Используйте эти скорости для расчета ускорения тележки в каждом случае.

  5. Занесите в таблицу массу и соответствующие значения ускорения, рассчитанные для каждого случая. Убедитесь, что каждый столбец и строка в таблице помечены соответствующим образом. 9{-2}$}\) по оси Y и \(\text{1}\) \(\text{см}\) = \(\text{1}\) \(\text{kg}\) по оси X.

  6. Используйте свой график для определения ускорения тележки, если ее масса равна \(\text{5}\) \(\текст{кг}\).

  7. Запишите заключение эксперимента.

Вы заметили в предыдущем исследовании, что чем тяжелее тележка, тем медленнее она движется, когда сила была постоянной. Ускорение обратно пропорционально массе. В математическом условия:

\(a\propto \frac{1}{m}\)

В подобном исследовании, когда масса поддерживается постоянной, а приложенная сила варьируется, вы обнаружить, что чем больше сила, тем быстрее будет двигаться объект. Ускорение тележки равно поэтому прямо пропорционально равнодействующей силе. В математических терминах:

\(а\пропто Ф.\)

Переставляя приведенные выше уравнения, мы получаем \(\propto\)\(\frac{F}{m}\) или \(F = ma\).

Помните, что и сила, и ускорение являются векторными величинами. Ускорение такое же направление как приложенная сила. Если несколько сил действуют одновременно, то мы только нужно работать с равнодействующей силой или чистой силой.

Второй закон движения Ньютона

Если на тело действует равнодействующая сила, оно заставит тело двигаться с ускорением в направлении Равнодействующая сила. Ускорение тела будет прямо пропорционально равнодействующей силой и обратно пропорциональна массе тела. Математическое представление равно: \[\vec{F}_{net} = m\vec{a}\]

Сила является векторной величиной . Второй закон Ньютона следует применить к \(y\)- и \(x\)-направления отдельно. Вы можете использовать результирующие \(y\)- и \(x\)-направления чтобы вычислить общую результирующую, как мы видели в предыдущей главе.

Видео: 23HC

temp text
Применение второго закона движения Ньютона

Второй закон Ньютона можно применять в самых разных ситуациях. Мы рассмотрим основные виды примеры, которые вам нужно изучить.

Рабочий пример 9: Второй закон Ньютона: Коробка на поверхности

Коробка \(\text{10}\) \(\text{kg}\) стоит на столе. Горизонтальная сила величины \(\text{32}\) \(\text{N}\) применяется к блоку. Сила трения величины \(\text{7}\) \(\text{N}\) присутствует между поверхностью и коробкой.

  1. Нарисуйте диаграмму сил, указав все силы, действующие на коробку.

  2. Рассчитайте ускорение коробки.

Определите горизонтальные силы и начертите диаграмму сил

Мы рассматриваем только силы, действующие в горизонтальном направлении (влево-вправо), а не в вертикальном направлении (вверх-вниз) силы. Приложенная сила и сила трения будут включены. Сила гравитация, являющаяся вертикальной силой, не будет учитываться.

Вычислите ускорение ящика

Помните , что мы рассматриваем направления \(y\) и \(x\) отдельно. В этой проблемы мы можем игнорировать \(y\)-направление, потому что коробка лежит на столе с гравитационная сила уравновешивается нормальной силой.

Нам дано:

Приложенная сила \({F}_{1}=\text{32}\text{ N}\)

Сила трения \({F}_{f}=-\text {7}\text{ Н}\)

Масса \(m=\text{10}\text{ кг}\)

Для расчета ускорения коробки мы будем использовать уравнение \(\vec{F}_{R}=m\vec{a}\). Поэтому: \begin{align*} \vec{F}_R &= m\vec{a} \\ \vec{F}_1+\vec{F}_f &= (\text{10})\vec{a} \\ (\text{32}-\text{7}) &= (\text{10})\vec{a} \\ \text{25} &= (\text{10})\vec{a} \\ \vec{a} &= \text{2,5}\text{ m·s$^{-2}$}\ \text{влево. } \end{выравнивание*} 9{-2}$}\) вправо. Одна треть всей силы трения действует на блоке \(\text{10}\) \(\text{kg}\) и две трети на блоке \(\text{15}\) \(\text{kg}\) блокировать. Рассчитать:

  1. величина и направление имеющейся общей силы трения.

  2. величина натяжения каната при Т.

Важно: когда у вас есть натяжение веревки в такой задаче, вы нужно знать, что оба конца веревки прилагают силу с одинаковой величина , но противоположное направление . Мы называем эту силу напряжения, и вы должны изучить силовые диаграммы в этой задаче внимательно .

Оцените то, что дано

Чтобы упростить задачу, давайте назовем два ящика метками, назовем \(\text{10}\) \(\text{kg}\) ящик номер 2 и \(\text{15}\) \(\text{kg}\) ящик номер 1.

У нас есть два ящика, общее ускорение которых равно данный. Тот факт, что ящики связаны веревкой, значит, они оба будут иметь одинаковое ускорение. Они также будут чувствовать одинаковую силу из-за натяжения веревки.

Нам говорят, что есть трение, но нам дают только соотношение между общим сила трения, которую испытывают оба ящика, и доля, которую испытывает каждый из них. Общая трение, \(\vec{F}_{fT}\) будет суммой трения на ящике 1, \(\vec{F}_{f1}\), и трение на ящике 2, \(\vec{F}_{f2}\). Нам говорят, что \(\vec{F}_{f1}=\frac{\text{2}}{\text{3}}\vec{F}_{fT}\) и \(\vec{F}_{f2}=\frac{\text{1}}{\text{3}}\vec{F}_{fT}\). Мы знаем, что блоки ускоряются вправо, и мы знаем, что трение будет в направлении, противоположном направлению движение и параллельно поверхности.

Диаграммы силы вытягивания

Диаграмма для ящика 1 будет:

Диаграмма для ящика 1 (обозначена синими пунктирными линиями) будет:

Где:

  • \(\vec{F}_{g1}\ ) — сила тяжести на первом ящике 90 124
  • \(\vec{N}_{1}\) – нормальная сила от поверхности на первый ящик
  • \(\vec{T}\) сила натяжения веревки
  • \(\vec{F}_{applied}\) — внешняя сила, приложенная к ящику
  • \(\vec{F}_{f1}\) – сила трения на первом ящике

Диаграмма для ящика 2 (обозначена оранжевыми пунктирными линиями) будет:

Где:

  • \(\vec{F}_{g2}\) – сила тяжести на втором ящике
  • \(\vec{N}_{2}\) – нормальная сила от поверхности на втором ящике
  • \(\vec{T}\) сила натяжения веревки
  • \(\vec{F}_{f2}\) – сила трения на втором ящике

Применить второй закон движения Ньютона

Задача говорит нам, что ящики ускоряются в направлении \(x\), что означает что силы в направлении \(y\) не приводят к результирующей силе. Мы можем лечить разных направлениях по отдельности, поэтому нам нужно учитывать только \(x\)-направление.

Мы работаем с одним измерением и можем выбрать соглашение о знаках для указания направления векторов. Мы выбираем векторы вправо (или в положительном \(x\)-направлении), чтобы они были положительный.

Теперь мы можем применить второй закон Ньютона к первому ящику, потому что мы знаем ускорение и мы знаем все силы, действующие на ящик. Использование положительного для обозначения сила вправо, мы знаем, что \({F}_{res1} = F_{applied}-{F}_{f1}-T\) \начать{выравнивать*} \vec{F}_{res1} &=m_1\vec{a} \\ F_{применяется}-{F}_{f1}-T &=m_1a \\ F_{применяется}-\frac{\text{2}}{\text{3}}{F}_{fT}-T &=m_1a \\ (500)-\frac{\text{2}}{\text{3}}{F}_{fT}-T &=(\text{15})(2) \\ -T &=(\text{15})(2) -(500)+\frac{\text{2}}{\text{3}}{F}_{fT} \end{выравнивание*}

Теперь применим второй закон Ньютона ко второму ящику, потому что мы знаем ускорение и мы знаем все силы, действующие на ящик. Мы знаем, что \({F}_{res2} = T-{F}_{f2}\). Обратите внимание, что натяжение в противоположном направлении. \начать{выравнивать*} \vec{F}_{res2} &=m_2\vec{a} \\ Т-{F}_{f2} &=m_2a \\ T – \frac{\text{1}}{\text{3}}{F}_{fT} &=m_2a \\ T &=(\text{10})(2) +\frac{\text{1}}{\text{3}}{F}_{fT} \конец{выравнивание*}

Решить одновременно

Мы использовали второй закон движения Ньютона, чтобы составить два уравнения с двумя неизвестными, это означает, что мы можем решить одновременно. Мы решили для \(T\) в приведенных выше уравнениях, но один имеет отрицательный знак, поэтому, если мы сложим два уравнения, мы вычтем значение напряжение, позволяющее нам решить для \({F}_{fT}\): \начать{выравнивать*} (T) + (-T) &= ((\text{10})(2) +\frac{\text{1}}{\text{3}}{F}_{fT}) + ((\ текст{15})(2) -(500)+\frac{\text{2}}{\text{3}}{F}_{fT}) \\ 0 & = \text{20} + \text{30} – \text{500} + \frac{\text{1}}{\text{3}}{F}_{fT} + \frac{\text{2}}{\text{3}}{F}_{fT} \\ 0 & = -\text{450} + {F}_{fT} \\ {F}_{fT} & = \text{450}\text{N} \end{выравнивание*}

Мы можем подставить величину \({F}_{fT}\) в уравнение для ящика 2, чтобы определить величина напряжения: \начать{выравнивать*} T &=(\text{10})(2) +\frac{\text{1}}{\text{3}}{F}_{fT} \\ T &=(\text{10})(2) +\frac{\text{1}}{\text{3}}(\text{450}) \\ Т &= \текст{20} +\текст{150} \\ Т &= \текст{170}\текст{ N} \end{выравнивание*}

Цитировать окончательные ответы

Суммарная сила трения равна \(\text{450}\) \(\text{N}\) влево. {-2}$}\) вправо. Одна треть всей силы трения действует на блоке \(\text{10}\) \(\text{kg}\) и две трети на блоке \(\text{15}\) \(\text{kg}\) блокировать. Рассчитать:

  1. величина и направление имеющейся общей силы трения.

  2. величина натяжения каната при Т.

Важно: когда у вас есть натяжение веревки в такой задаче, вы нужно знать, что оба конца веревки прилагают силу с то же величина , но противоположное направление . Мы называем эту силу напряжения, и вы должны изучить силовые диаграммы в этой задаче осторожно .

Нарисуйте диаграмму сил

Всегда рисуйте диаграмму сил, даже если вопрос не требует этого. Ускорение дана вся система, поэтому будет построена силовая диаграмма всей системы. Поскольку два ящика рассматриваются как единое целое, диаграмма сил будет выглядеть так: 9{-2}$}\)). Выберите направление движения в качестве положительного направления (к справа положительный).

\начать{выравнивать*} {F}_{R} & = ма\\ {F} _ {\ text {применяется}} + {F} _ {f} & = ma \\ \text{500}+{F}_{f} & =\влево(10+15\вправо)\влево(2\вправо)\\ {F}_{f} & =50-\text{500}\\ {F}_{f} & =-\text{450}N \конец{выравнивание*}

Сила трения \(\text{450}\) \(\text{N}\) противоположна направлению движения (к слева).

Найти натяжение в веревке

Чтобы найти натяжение в веревке нам нужно заглянуть в один из двух ящиков самостоятельно. Давайте выберите ящик \(\text{10}\) \(\text{kg}\). Во-первых, нам нужно нарисовать диаграмму сил:

Рисунок 2.5: Диаграмма усилия ящика \(\text{10}\) \(\text{kg}\).

Сила трения на блоке \(\text{10}\) \(\text{kg}\) составляет одну треть от общей, следовательно:

\({F}_{f}=\frac{\text{1}}{\text{3}}\times \text{450}\)

\({F}_{f} =\text{150} \text{N}\)

Если применить второй закон Ньютона:

\начать{выравнивать*} {F}_{R}& = ма \\ T+{F}_{f}& = \влево(10\вправо)\влево(2\вправо) \\ T+\влево(-\текст{150}\вправо)& = 20 \\ T& = \text{170}\text{N} \конец{выравнивание*}

Примечание. Если бы мы использовали тот же принцип и применили его к ящику \(\text{15}\) \(\text{kg}\), наши расчеты были бы следующими:

\начать{выравнивать*} {F}_{R}& = ма \\ {F} _ {\ text {применяется}} + T + {F} _ {f} & = \ влево (15 \ вправо) \ влево (2 \ вправо) \\ \text{500}+T+\влево(-\text{300}\вправо)& = 30 \\ Т& = -\текст{170}\текст{N} \конец{выравнивание*}

Отрицательный ответ здесь означает, что сила действует в направлении, противоположном движению, в другими словами влево, что правильно. Однако возникает вопрос о величине сила и ваш ответ будут заключены в кавычки \(\text{170}\) \(\text{N}\).

Рабочий пример 12: Второй закон Ньютона: человек тянет коробку

Мужчина тянет ящик \(\text{20}\) \(\text{kg}\) за веревку, образующую угол \(\text{60}\)\(\text{°}\) по горизонтали. Если он применит силу величины \(\text{150}\) \(\text{N}\) и сила трения величины \(\text{15}\) \(\text{N}\) присутствует, рассчитайте ускорение коробки.

Нарисуйте диаграмму сил или диаграмму свободного тела

Движение горизонтальное, поэтому мы будем рассматривать силы только в горизонтальном направлении. направление. Помните, что вертикальные силы не влияют на горизонтальное движение и наоборот.

Вычислите горизонтальную составляющую приложенной силы

Сначала нам нужно выбрать положительное направление в этой задаче. Мы выбрали положительное \(х\)-направление (вправо) должно быть положительным.

Приложенная сила действует под углом \(\text{60}\)\(\text{°}\) к горизонтали. Мы можем рассматривать только силы, параллельные движению. Горизонтальная составляющая приложенная сила должна быть рассчитана, прежде чем мы сможем продолжить: \начать{выравнивать*} F_x & = F_{применяется}\cos(\theta) \\ &= \text{150}\cos(\text{60}\text{°}) \\ & = \текст{75}\текст{N} \end{выравнивание*} 9{-2}$}\) вправо.

Рабочий пример 13: второй закон Ньютона: грузовик и прицеп

Грузовик \(\text{2 000}\) \(\text{кг}\) тянет за собой прицеп \(\text{500}\) \(\text{кг}\) с постоянное ускорение. Двигатель грузовика развивает тягу \(\text{10 000}\) \(\текст{N}\). Не учитывать влияние трения. Вычислите:

  1. ускорение грузовика; и

  2. натяжение тягово-сцепного устройства Т между грузовиком и прицепом, если тягово-сцепное устройство делает угол \(\text{25}\)\(\text{°}\) с горизонтом.

Рисунок 2.6: Грузовик тянет прицеп.

Начертить силовую диаграмму

Нарисуйте диаграмму сил с указанием всех сил, действующих на систему в целом:

Рисунок 2. 7: Бесплатные схемы кузова для грузовика с прицепом.

Применить второй закон движения Ньютона

Мы выбираем положительное направление \(x\) как положительное направление. Нам нужно только рассмотреть горизонтальные силы. Использование только горизонтальных сил означает, что мы сначала должны отметить что натяжение действует под углом к ​​горизонтали и нам нужно использовать горизонтальную составляющая напряженности в наших расчетах.

Горизонтальная составляющая имеет величину \(T\cos(\text{25}\text{°})\).

При отсутствии трения единственной силой, вызывающей ускорение системы, является тяга двигателя. Если мы теперь применим второй закон движения Ньютона к грузовику, мы получим: \начать{выравнивать*} \vec{F}_{Rtruck} &= m_{truck}\vec{a}\ \text{(мы используем знаки для указания направления)} \\ {F}_{engine} – T\cos(\text{25}\text{°}) &= (\text{2 000})a \\ (\text{10 000}) – T\cos(\text{25}\text{°}) &= (\text{2 000})a \\ a & = \frac{(\text{10 000}) – T\cos(\text{25}\text{°})}{(\text{2 000})} \end{выравнивание*}

Теперь применим тот же принцип к прицепу (помните, что направление натяжения будет противоположно случаю грузовика): \начать{выравнивать*} \vec{F}_{Rtrailer} &= m_{trailer}\vec{a}\ \text{(мы используем знаки для указания направления)} \\ T\cos(\text{25}\text{°}) &= (\text{500})a \\ a & = \frac{T\cos(\text{25}\text{°})}{(\text{500})} \end{выравнивание*}

Теперь у нас есть два уравнения и два неизвестных, поэтому мы можем решать их одновременно. Мы вычитаем второе уравнение от первого, чтобы получить: \начать{выравнивать*} (а) – (а) &=(\frac{(\text{10 000}) – T\cos(\text{25}\text{°})}{(\text{2 000})}) – (\frac{T\cos(\text{25}\text{°})}{(\text{500})}) \\ 0 & = (\frac{(\text{10 000}) – T\cos(\text{25}\text{°})}{(\text{2 000})}) – (\frac{T\cos(\text{25}\text{°})}{(\text{500})})\\ & \text{(умножить на \text{2 000})} \\ 0 & = (\text{10 000}) – T\cos(\text{25}\text{°}) – 4T\cos(\text{25}\text{°}) \\ \text{5}T\cos(\text{25}\text{°}) & = (\text{10 000}) \\ T & = \frac{(\text{10 000})}{\text{5}\cos(\text{25}\text{°})} \\ T & = \text{2 206,76}\text{N} \end{выравнивание*} 9{-2}$} \конец{выравнивание*}

временный текст
Объект на наклонной плоскости

В предыдущем разделе мы рассмотрели компоненты гравитационной силы, параллельные и перпендикулярно склону для объектов на наклонной плоскости. Когда мы смотрим на проблемы с наклонной плоскости нам нужно включить составляющую силы тяжести, параллельную наклону.

Вспомните изображения книги на столе, когда одна сторона стола поднимается выше книги. начинает скользить. Почему? Книга начинает скользить, потому что составляющая гравитационной силы параллельно поверхности стола становится больше при большем угле наклона. Это как приложенной силы, и в конечном итоге она становится больше, чем сила трения и книга ускоряется вниз по столу или наклонной плоскости.

Сила гравитации также стремится толкнуть объект «в» склон. Это составляющая усилие перпендикулярно склону. Движения в этом направлении нет, так как эта сила уравновешена наклоном, прижимающимся к объекту. Эта «толкающая сила» является нормальной силой (Н). о которой мы уже узнали и равна по величине перпендикулярной составляющей сила тяжести, но противоположная по направлению.

Не используйте аббревиатуру \(W\) для веса, так как она используется для сокращения «работа». Скорее используйте сила тяжести \({F}_{g}\) для веса.

Моделирование: 23HD

Рабочий пример 14: Второй закон Ньютона: коробка на наклонной плоскости

Тело массой \(M\) покоится на наклонной плоскости за счет трения.

Какой из следующих вариантов является величиной силы трения, действующей на тело?

  1. \(F_g\)

  2. \(F_g\cos(θ)\)

  3. \(F_g\sin(θ)\)

  4. \(F_g\tan(θ)\)

Проанализируйте ситуацию

Вопрос просит нас определить величину силы трения. Говорят, что тело находиться в покое на плоскости, а это значит, что она не движется, и поэтому ускорение равно нуль. Мы знаем, что сила трения будет действовать параллельно склону. Если бы не было трения коробка будет скользить вниз по склону, поэтому трение должно действовать вверх по склону. Мы также знать, что будет составляющая силы тяжести, перпендикулярная склону и параллельная склон. Диаграмма свободного тела для сил, действующих на блок:

Определить величину силы трения

К этой задаче можно применить второй закон Ньютона. Мы знаем, что объект не движется, поэтому результирующее ускорение равно нулю. Мы выбираем вверх по склону, чтобы быть положительным направлением. Следовательно: \начать{выравнивать*} \vec{F}_R &= m\vec{a}\; \text{используя знаки для направления}\\ F_f – F_g\sin(\theta) &= m (0)\\ F_f – F_g\sin(\theta) &= m (0)\\ F_f & = F_g\sin(\тета) \end{выравнивание*}

Приведите свой окончательный ответ

Сила трения имеет ту же величину, что и составляющая силы гравитации параллельно склону, \(F_g\sin(\theta)\).

Рабочий пример 15: Второй закон Ньютона: объект на наклонной плоскости

Сила величины \(T=\text{312} \text{N}\) вверх по склону требуется, чтобы удержать тело в состоянии покоя на наклонной плоскости без трения, которая составляет угол \(\text{35}\)\(\text{°}\) по горизонтали. Вычислите величины силы из-за гравитации и нормальной силы, давая ваши ответы с тремя значащими цифрами.

Найдите величину \(\vec{F}_g\)

Обычно нас просят найти величину \(\vec{T}\), но в этом случае \(\vec{T}\) задано, и нас просят найти \(\vec{F}_g\). Мы можем использовать то же уравнение. \(T\) является сила, которая уравновешивает компонент \(\vec{F}_g\), параллельный плоскости (\({F}_{gx}\)) и поэтому он имеет одинаковую величину.

К этой задаче можно применить второй закон Ньютона. Мы знаем, что объект не движется, поэтому результирующее ускорение равно нулю. Мы выбираем вверх по склону, чтобы быть положительным направлением. Следовательно: \начать{выравнивать*} \vec{F}_R &= m\vec{a}\; \text{используя знаки для направления}\\ T – F_g\sin(\theta) &= m (0)\\ F_g & = \frac{T}{\sin(\theta)} \\ & = \frac{\text{312}}{\sin(\text{35}\text{°})} \\ & = \текст{543,955}\text{ Н} \конец{выравнивание*}

Найдите величину \(\vec{N}\)

Мы рассматриваем силы, параллельные и перпендикулярные наклону, отдельно. Блок неподвижен, поэтому ускорение перпендикулярно склону равно нулю. Еще раз можем подать заявку Второй закон Ньютона. Выбираем направление нормальной силы как положительное направление. \начать{выравнивать*} \vec{F}_R &= m\vec{a}\; \text{используя знаки для направления}\\ N – F_g\cos(\theta) &= m (0)\\ N & = F_g\cos(\тета) \end{выравнивание*}

Мы могли бы подставить значение \(F_g\), вычисленное ранее. Мы хотели бы проиллюстрировать что есть еще один подход, который поможет вам получить правильный ответ, даже если вы ошибся в расчете \(F_g\). \(F_g\cos(\theta)\) также можно определить с помощью тригонометрических соотношений. Мы знаем из предыдущей части вопроса, что \(T = F_g\sin(\theta)\). Мы также знаем, что \начать{выравнивать*} \tan(\theta) &=\frac{F_g\sin(\theta)}{F_g\cos(\theta)}\\ &=\фракция{T}{N}\\ N&=\frac{T}{\tan(\theta)}\\ & = \ frac {\ text {312}} {\ tan (\ text {35} \ text {°})} \\ & = \text{445,58}\text{N} \end{выравнивание*}

Обратите внимание, что в вопросе предлагается давать ответы с тремя значащими цифрами. Поэтому мы округлить \(\vec{N}\) от \(\text{445,58}\) \(\text{N}\) до \(\text{446}\) \(\text{N}\ ) перпендикулярно поверхности вверх и \(\vec{T}\) от \(\text{543,955}\) \(\text{N}\) вверх до \(\text{544}\) \(\text{N}\) параллельно плоскости вверх по склону.

временный текст
Лифты и ракеты

До сих пор мы рассматривали объекты, которые тянут или толкают по поверхности, другими словами, движение. параллельно поверхности, на которой лежит предмет. Здесь мы рассмотрели только силы, параллельные поверхности, но мы также можем поднимать предметы или позволять им падать. Это вертикальное движение, при котором действуют только вертикальные силы. рассматриваются.

Рассмотрим лифт \(\text{500}\) \(\text{kg}\) без пассажиров, подвешенный на тросе. цель троса – тянуть лифт вверх, чтобы он мог добраться до следующего этажа или опустить поднять так, чтобы он мог двигаться вниз на этаж ниже. Мы рассмотрим пять возможных этапов во время движение лифта и применить наше знание второго закона движения Ньютона к ситуации. 5 этапов:

  1. Стационарный подъемник, подвешенный над землей.
  2. Лифт ускоряется вверх.
  3. Лифт движется с постоянной скоростью.
  4. Лифт тормозит (тормозится).
  5. Лифт, ускоряющийся вниз (трос рвется!).

Мы выбрали восходящее направление в качестве положительного направления для этого обсуждения.

Этап 1:

Лифт \(\text{500}\) \(\text{kg}\) стоит на втором этаже высотного здания.

Лифт не ускоряется. Должно быть натяжение \(\vec{T}\) троса, действующего на подъемник и должна быть сила тяжести, \(\vec{F}_g\). Других сил нет, и мы можно нарисовать схему свободного тела:

Применим второй закон Ньютона к вертикальному направлению: \начать{выравнивать*} \vec{F}_R & = m_{\text{лифт}} \vec{a}\ \text{(мы используем знаки для указания направления)} \\ T – F_g & = m _ {\ text {подъем}} (0) \\ Т & = F_g \end{выравнивание*} 9{-2}$}\).

Если лифт ускоряется, это означает, что в направлении движения действует равнодействующая сила. Это означает, что сила, действующая вверх, теперь больше, чем сила тяжести \(\vec{F}_g\) (вниз). Чтобы найти величину \(\vec{T}\), применяемую кабелем, мы можем сделать следующее. расчет: (Помните, что мы выбрали вверх как положительное значение.)

Применим второй закон Ньютона к вертикальному направлению: \начать{выравнивать*} \vec{F}_R & = m_{\text{лифт}} \vec{a}\ \text{(мы используем знаки для указания направления)} \\ T – F_g & = m _ {\ text {подъемник}} (\ text {1}) \\ T & = F_g + m _ {\ text {подъем}} (\ text {1}) \end{выравнивание*}

Ответ имеет смысл, так как нам нужна большая сила вверх, чтобы нейтрализовать эффект гравитации, а также иметь положительную результирующую силу.

Этап 3:

Лифт движется с постоянной скоростью.

Когда лифт движется с постоянной скоростью, ускорение равно нулю, \начать{выравнивать*} \vec{F}_R & = m_{\text{лифт}} \vec{a}\ \text{(мы используем знаки для указания направления)} \\ T – F_g & = m _ {\ text {подъем}} (0) \\ Т & = F_g \end{выравнивание*} 9{-2}$}\). Лифт двигался вверх так что это означает, что он замедляется или ускоряется в направлении, противоположном направлению движения. движение. Это означает, что ускорение имеет отрицательное направление. \начать{выравнивать*} \vec{F}_R & = m_{\text{лифт}} \vec{a}\ \text{(мы используем знаки для указания направления)} \\ T – F_g & = m _ {\ text {подъем}} (- \ text {2}) \\ T & = F_g – \ text {2} m _ {\ text {подъем}} \end{выравнивание*}

Поскольку лифт теперь замедляется, результирующая сила направлена ​​вниз. Это означает, что сила сила, действующая вниз, больше силы, действующей вверх.

Это имеет смысл, так как нам нужна меньшая сила, направленная вверх, чтобы результирующая сила была направлена ​​вниз. Сила тяжести теперь больше, чем тяга троса вверх, и лифт замедлится.

Этап 5:

Трос обрывается.

Когда трос обрывается, силы, которая раньше действовала вверх, больше нет. Единственная сила то, что присутствует, было бы силой тяжести. Лифт будет падать свободно и его ускорение.

Кажущийся вес

Ваш вес – это величина гравитационной силы, действующей на ваше тело. Когда вы стоите в лифте то есть канцтовары а потом начинает ускоряться вверх чувствуешь что тебя вдавливают в пол пока лифт разгоняется. Вы чувствуете, что вы тяжелее, и ваш вес больше. Когда вы находитесь в канцелярский подъемник, который начинает разгоняться вниз, вы чувствуете себя легче на ногах. Вы чувствуете себя ваш вес меньше.

Вес измеряется нормальными силами. Когда лифт ускоряется вверх, вы чувствуете себя более нормальным. сила, действующая на вас как сила, необходимая для ускорения вас вверх в дополнение к уравновешиванию сила гравитации.

Когда лифт ускоряется вниз, вы чувствуете меньшую нормальную силу, действующую на вас. Это потому, что чистая сила вниз требуется, чтобы ускорить вас вниз. Это явление называется кажущийся вес потому что ваш вес на самом деле не изменился.

временный текст
Ракеты

Как и в случае с лифтами, ракеты также являются примерами объектов в вертикальном движении. Сила тяжести тянет ракета вниз, а тяга двигателя толкает ракету вверх. Сила, с которой двигатель сила тяжести должна преодолеть силу тяжести, чтобы ракета могла двигаться вверх. Работал В приведенном ниже примере рассматривается применение второго закона Ньютона при запуске ракеты. 9{-2}$}\) вверх.

\(\vec{F}_g\)=\(\text{49 000}\) \(\text{N}\) вниз.

Нас просят найти тягу ракетного двигателя \(\vec{F}\).

Найдите тягу двигателя

Применим второй закон Ньютона: \начать{выравнивать*} \vec{F}_R & = m\vec{a}\ \text{(используя знаки для указания направления)} \\ F – F_g & = (\text{5 000})(\text{20}) \\ Ф – (\текст{49000}) & = (\text{5 000})(\text{20}) \\ F &= \text{149 000}\text{ N} \конец{выравнивание*}

Процитируйте свой окончательный ответ

Сила тяги направлена ​​\(\text{149 000}\) \(\text{N}\) вверх.

Рабочий пример 17: Ракеты

Как ракеты разгоняются в космосе?

  • Внутри ракеты взрывается газ.

  • Этот взрывной газ воздействует на каждую сторону ракеты. (как показано на рисунке ниже взрывной камеры внутри ракета).

  • Из-за симметрии положения все приложенные силы на ракете уравновешиваются силами противоположной стороны, кроме силы напротив открытой стороны. Эта сила на верхней поверхности неуравновешена.

  • Следовательно, это результирующая сила, действующая на ракету, и она заставляет ракету двигаться вперед.

Учебник Упражнение 2.5

Буксир способен тянуть корабль с силой \(\text{100}\) \(\text{кН}\). Если два таких буксира тянут за один корабль, они могут создавать любую силу в пределах от от \(\text{0}\) \(\text{кН}\) до максимального \(\text{200}\) \(\text{кН}\). Дайте подробное объяснение, как это возможно. Используйте диаграммы, чтобы поддержать ваши результат.

Начнем с того, что два буксира тянут в противоположных направлениях:

Результирующая сила равна \(\text{0}\) \(\text{кН}\), так как буксиры тянут с равными силами в противоположных направлениях.

Если два буксира тянут в одном направлении, то получим:

Результирующая сила равна \(\text{200}\) \(\text{кН}\), так как буксиры тянут с равными силами в одном направлении. {-2}$} \end{выравнивание*} 9{-2}$}\).

\begin{выравнивание*} F & = ма \\ m & = \frac{F}{a} \\ & = \ гидроразрыва {\ текст {40}} {\ текст {2}} \\ & = \text{20}\text{ кг} \end{align*}

Найдите ускорение тела массой \(\text{1 000}\) \(\text{kg}\), которое сила с величиной \(\text{150}\) \(\text{N}\), действующая на него. 9{-2}$}\) силой величины \(\text{25}\) \(\text{N}\).

\begin{выравнивание*} F & = ма \\ m & = \frac{F}{a} \\ & = \ гидроразрыва {\ текст {25}} {\ текст {3}} \\ & = \text{8,33}\text{ кг} \end{align*}

Определить ускорение массы \(\text{24}\) \(\text{кг}\) при воздействии силы величина \(\text{6}\) \(\text{N}\) действует на него. Чему равно ускорение, если сила удвоилась, а масса уменьшилась вдвое? 9{-2}$}\).

Вы можете проверить это, удвоив силу и уменьшив вдвое массу.

Определите результирующую силу, вызывающую ускорение.

\begin{выравнивание*} F & = ма \\ & = (\текст{8})(\текст{5}) \\ & = \текст{40}\текст{N} \end{align*}

Какое ускорение возникло бы, если бы мы удвоили силу и уменьшили массу на половину? 9{-2}$} \end{align*}

С какой скоростью он будет двигаться после \(\text{20}\) \(\text{s}\)?

Мы можем использовать уравнения движения (вспомним из 10 класса: движение в одном размер), чтобы определить, как быстро он будет двигаться:

\начать{выравнивать*} v_{f} & = v_{i} + at \\ & = 0 + (5)(20) \\ & = \text{100}\text{ м·с$^{-1}$} \конец{выравнивание*} 9{-1}$}\)?

Опять же, мы можем использовать уравнения движения (вспомним из 10 класса: движение в одном размер), чтобы определить, сколько времени потребуется для достижения заданной скорости:

\начать{выравнивать*} v_{f} & = v_{i} + at \\ 35 & = 0 + (5)t \\ т & = \ гидроразрыва {\ текст {35}} {\ текст {5}} \\ & = \текст{7}\текст{ы} \конец{выравнивание*} 9{2} \\ & = \text{562,5}\text{м} \конец{выравнивание*}

Рассчитать составляющую силы \(\text{200}\) \(\text{N}\), которая ускоряет блок горизонтально.

\begin{выравнивание*} F_{x} & = F \cos\тета\\ & = (\текст{200}) \cos(60) \\ & = \текст{100}\текст{N} \end{выравнивание*} 9{-2}$}\), рассчитать величину силы трения на бруске.

\begin{выравнивание*} F_{R} & = ма \\ F_{x} + F_{f} & = ma \\ \text{100} + F_{f} & = (50)(\text{1,5}) \\ F_{f} & = \text{75} – \text{100} \\ & = -\текст{25}\текст{N} \end{выравнивание*}

Рассчитайте вертикальную силу, действующую на плоскость блока.

\begin{выравнивание*} F_{y} & = F \sin\theta\\ & = (\текст{200}) \sin(60) \\ & = \text{173,2}\text{N} \end{align*}

Игрушечная ракета испытывает гравитационную силу \(\text{4,5}\) \(\text{N}\) поддерживается вертикально, помещая его в бутылку. {-2}$}\).

Принятие положительного направления вверх:

\начать{выравнивать*} F_{R} & = ма \\ F_{1} + F_{g} & = ма \\ F_{1} – \text{4,5} & = (\text{0,5})(\text{8}) \\ F_{1} & = \text{4} + \text{4,5} \\ & = \текст{8,5}\текст{N} \конец{выравнивание*}

Постоянная сила величины \(\text{70}\) \(\text{N}\) приложена вертикально к блок, как показано. На блок действует сила тяжести \(\text{49}\) \(\текст{N}\). Вычислите ускорение блока.

Положительное направление вверх:

\начать{выравнивать*} F_{R} & = ма \\ F_{1} + F_{g} & = ма \\ \текст{70} – \текст{49{-2}$} \конец{выравнивание*}

Лифт движется вверх или вниз? Обоснуйте свой ответ. {-2}$}\). Сила трения \(\text{700}\) \(\text{N}\) препятствует его движению. Какую силу развивает двигатель автомобиля?

\begin{выравнивание*} \vec{F}_{R} & = m\vec{a} \\ \vec{F}_{f} + \vec{F}_{E} & = m\vec{a} \\ -\text{700} + F_{E} & = (\text{800})(4) \\ F_{E} & = \text{3 200} + \text{700} \\ & = \text{3 900}\text{ N} \end{align*}

Два объекта массой \(\text{1}\) \(\text{kg}\) и \(\text{2}\) \(\text{kg}\ ) соответственно, укладываются на гладкую поверхность и соединяются ниткой. А горизонтальная сила \(\text{6}\) \(\text{N}\) приложена с помощью пружины баланс на объект \(\text{1}\) \(\text{kg}\). Игнорируя трение, что будет сила, действующая на массу \(\text{2}\) \(\text{kg}\), измеренная второй пружиной баланс, быть?

Сила, действующая на блок \(\text{2}\) \(\text{kg}\), равна \(\text{6}\) \(\текст{N}\). Поскольку предполагается, что поверхность не имеет трения, приложенная к ней сила блок \(\text{1}\) \(\text{kg}\) равен силе, которую испытывает \(\text{2}\) \(\text{kg}\) блок.

Каково его ускорение на Земле, где на него действует гравитационная сила из \(\text{1 960}\) \(\text{N}\)?

Сила, действующая на ракету, направлена ​​вверх, а сила, сила тяжести направлена ​​вниз. Принятие вверх как положительное:

\начать{выравнивать*} F_{R} & = ма \\ F_ {g} + F _ {\ text {ракета}} & = ma \\ -\text{1 960} + \text{4 000} & = ма \\ а & = \ гидроразрыва {\ текст {2 040}} {\ текст {200}} \\ & = \text{10,2}\text{ м·с$^{-2}$} \конец{выравнивание*}

Какую движущую силу ракетный двигатель должен воздействовать на заднюю часть ракеты на земле?

На Земле ракетные двигатели должны преодолевать силу гравитации и поэтому должны прилагать силу \(\text{1 960}\) \(\text{N}\) или больше. {-2}$} \конец{выравнивание*}

Если автомобиль весит \(\text{1 000}\) \(\text{кг}\), какое усилие действуют на тормоза прилагать?

\begin{выравнивание*} F & = ма \\ & = (\текст{1 000})(\текст{10}) \\ & = \text{10 000}\text{ N} \end{align*}

На брусок на наклонной плоскости действует сила тяжести, \(\vec{F}_g\) \(\text{300}\) \(\text{N}\) прямо вниз. Если склон имеет уклон \(\text{67,8}\)\(\text{°}\) к горизонтали, какова составляющая сила тяжести перпендикулярна и параллельна склону? Под каким углом будет перпендикулярная и параллельная составляющие силы тяжести равны?

Компонент, параллельный уклону:

\начать{выравнивать*} F_{gx} & = F \sin\theta\\ & = (\текст{300})\sin(\текст{67,8}) \\ & = \текст{277,76}\текст{N} \конец{выравнивание*}

Компонент, перпендикулярный уклону:

\начать{выравнивать*} F_{gy} & = F \cos\theta\\ & = (\текст{300})\cos(\текст{67,8}) \\ & = \текст{113,35}\текст{N} \конец{выравнивание*}

Чтобы два компонента были равны, угол должен быть равен \(\text{45}\)\(\text{°}\). (\(\sin (45) = \cos(45)\)).

Блок на наклонной плоскости подвергается действию силы тяжести, \(\vec{F}_g\) \(\text{287}\) \(\text{N}\) прямо вниз. Если составляющая гравитационного сила, параллельная наклону, равна \(\vec{F}_{gx}\)=\(\text{123,7}\) \(\text{N}\) в отрицательное \(х\)-направление (вниз по склону), каков наклон склона?

\begin{выравнивание*} F_{gx} & = F \sin\theta\\ \text{123,7} & = (\text{287})\sin \theta \\ \sin\theta&=\text{0,431}\ldots\\ \тета & = \текст{25,53}\текст{°} \end{align*}

На брусок на наклонной плоскости действует сила тяжести, \(\vec{F}_g\) \(\text{98}\) \(\text{N}\) прямо вниз. Если наклон наклонен в неизвестном направлении угла к горизонтали, но нам говорят, что отношение компонентов сила тяжести, перпендикулярная и параллельная склону, составляет 7:4. Что это угол наклона образует с горизонталью?

Сначала запишем уравнения для параллельной и перпендикулярной составляющих:

\начать{выравнивать*} F_{gx} & = F \sin\theta\\ & = (98)\sin\тета \конец{выравнивание*} \начать{выравнивать*} F_{gy} & = F \cos\theta\\ & = (98)\cos\тета \конец{выравнивание*}

Теперь отметим следующее:

\начать{выравнивать*} 7F_{gx} & = 4F_{gy} \\ \следовательно, 7(98)\sin\theta & = 4(98)\cos\theta \конец{выравнивание*}

Теперь нам нужно найти тета:

\начать{выравнивать*} \sin \theta & = \frac{\text{392}}{\text{686}} \cos \theta \\ & = \text{0,5714} \cos\тета\\ \ гидроразрыва {\ греха \ тета} {\ соз \ тета} & = \ текст {0,5714} \\ \загар \тета & = \текст{0,5714} \\ \тета & = \текст{25,53}\текст{°} \конец{выравнивание*}

Напомним из тригонометрии, что \(\frac{\sin \theta}{\cos \theta} = \tan \theta\). {-2}$}\)). Выберите направление движения, которое будет положительное направление (вправо положительное).

\начать{выравнивать*} F_{R} & = ма\\ F _ {\ text {применяется}} + F_ {f} & = ma \\ \text{1 500} + {F}_{f} & = (30+50)(2)\\ F_{f} & = \text{160} – \text{1 500}\\ F_{f} & = -\text{1 340}\text{N} \конец{выравнивание*}

величина натяжения каната при Т.

Заметим, что \(m_{1} = \frac{\text{5}}{\text{3}} m_{2}\), поэтому \ (F_{f1} = \frac{\text{5}}{\text{3}} F_{f2}\).

Чтобы найти натяжение веревки, нам нужно посмотреть на один из двух ящиков на их собственный. Давайте выберем ящик \(\text{30}\) \(\text{kg}\).

Сила трения на блоке \(\text{30}\) \(\text{kg}\) указана выше. Мы может вычислить \(F_{f2}\):

\начать{выравнивать*} F_{f} & = F_{f1} + F_{f2} \\ \text{1 340} & = F_{f2} + \frac{\text{5}}{\text{3}} F_{f2} \\ \text{1 340} & = \frac{\text{8}}{\text{3}} F_{f2} \\ F_{f2} & = \text{502,5}\text{N} \конец{выравнивание*}

Если применить второй закон Ньютона:

\начать{выравнивать*} F_{R} & = ма \\ T + F_{f} & = (30)(2) \\ Т + -\текст{502,5} & = 60 \\ T & = \text{562,5}\text{N} \конец{выравнивание*}

величина и направление общей силы трения. {2} + \frac{\text{9}{2} & = \text{63 555,88} \\ F_{gx1} & = \text{252,10}\text{N} \конец{выравнивание*}

Теперь мы можем написать выражение для результирующей силы, действующей на каждый ящик:

\начать{выравнивать*} \vec{F}_{R} & = ма \\ F_{A} – F_{gx1} – F_{f} – T & = ma \\ \text{500} – \text{420,17} – F_{f1} – T & = (\text{50})(\text{7})\\ -T & = \text{420,17} + F_{f} \конец{выравнивание*} \начать{выравнивать*} \vec{F}_{R} & = ма \\ T – F_{gx2} – F_{f} & = ma \\ T – \text{252,10} – F_{f} & = (\text{30})(\text{7})\\ T & = \text{462,1} + F_{f} \конец{выравнивание*}

И одновременно решить для силы трения:

\начать{выравнивать*} -T + T & = \text{420,17} + F_{f} + \text{462,1} + F_{f} \\ 0 & = \text{882,27} + 2F_{f} \\ F_{f} & = -\text{441,14}\text{N} \конец{выравнивание*}

Величина натяжения каната при Т.

Мы можем использовать любое из двух приведенных выше выражений, чтобы найти натяжение. Мы будем использовать выражение для ящика 1:

\начать{выравнивать*} -T & = \text{420,17} + F_{f} \\ & = \текст{420,17} + -\текст{441,14} \\ T & = \text{20,97}\text{N} \конец{выравнивание*}

Третий закон движения Ньютона (ESBKV)

Третий закон движения Ньютона касается взаимодействия между парами объектов. Например, если вы держите книгу у стены, вы прикладываете силу к книге (чтобы удержать ее там), и книга воздействуя на вас силой (чтобы вы не провалились сквозь книгу). Это может показаться странным, но если книга не отталкивала тебя, твоя рука проталкивала книгу! Эти две силы (сила силы руки на книгу (\({F}_{1}\)) и силу книги на руку (\({F}_{2}\))) называют пара сил действие-противодействие. Они имеют одинаковую величину, но действуют в противоположных направлениях и действуют на различные предметы (одна сила действует на книгу, а другая — на вашу руку).

В этой ситуации присутствует еще одна пара сил действие-противодействие. Книга выступает против стена (сила действия) и стена отталкивает книгу (противодействие). Сила книги на стена (\({F}_{3}\)) и сила воздействия стены на книгу (\({F}_{4}\)) показаны на диаграмме.

Рисунок 2.8: Пары действие-противодействие Ньютона.
Третий закон движения Ньютона

Если тело А действует на тело В с силой, то тело В действует на тело А с такой же силой, но в противоположном направлении.

Видео: 23J9

Эти пары действие-противодействие имеют несколько свойств:

  • один и тот же тип силы действует на объекты,
  • силы имеют одинаковую величину, но противоположное направление, и
  • силы действуют на разные объекты.

Ньютоновские пары действие-противодействие можно найти везде в жизни, где два объекта взаимодействуют с одним еще один. Следующие рабочие примеры иллюстрируют это:

Рабочий пример 18: третий закон Ньютона – ремень безопасности

Динео сидит на пассажирском сиденье автомобиля с пристегнутым ремнем безопасности. Машина внезапно останавливается и он движется вперед (первый закон Ньютона – он продолжает свое движение) до тех пор, пока ремень безопасности не останавливает его. Нарисуйте помеченную диаграмму силы, определяющую две пары действие-противодействие в этом ситуация.

Нарисуйте диаграмму силы

Начните с рисования рисунка. Вы будете использовать стрелки для обозначения сил, так что изображение достаточно большое, чтобы можно было добавить подробные метки. Картинка должна быть точно, но не художественно! Используйте человечков-палочек, если нужно.

Подпишите схему

Берите по одной паре и тщательно маркируйте их. Если на чертеже недостаточно места, затем используйте ключ сбоку.

Рабочий пример 19: Третий закон Ньютона: силы в лифте

Тэмми едет с первого этажа на пятый этаж отеля в лифте, движущемся с постоянной скоростью. скорость. Какое ОДНО из следующих утверждений о величине приложенной силы ВЕРНО на полу лифта на ногах Тэмми? Используйте третий закон Ньютона, чтобы обосновать свой ответ.

  1. Это больше, чем величина веса Тэмми.

  2. По величине она равна силе, с которой ноги Тэмми воздействуют на пол лифта.

  3. Он равен тому, что был бы в стационарном подъемнике.

  4. Это больше, чем в стационарном подъемнике.

Проанализируйте ситуацию

Это вопрос третьего закона Ньютона, а не второго закона Ньютона. Нам необходимо сосредоточиться на пары сил действие-противодействие, а не движение лифта. Следующая диаграмма покажет пары действие-противодействие, которые присутствуют, когда человек стоит на весах в лифте.

Рисунок 2.9: Ньютоновские пары действие-противодействие в подъемной силе.

В этом вопросе делается утверждение о силе пола (лифта) на ноги Тэмми. Этот сила соответствует \({F}_{2}\) на нашей диаграмме. Сила реакции, которая сочетается с этой равно \({F}_{1}\), то есть силе, с которой ноги Тэмми воздействуют на пол лифта. величины этих двух сил одинаковы, но они действуют в противоположных направлениях.

Выберите правильный ответ

Важно сначала проанализировать вопрос, прежде чем смотреть ответы. Ответы могут запутать вас, если вы посмотрите на них в первую очередь. Убедитесь, что вы понимаете ситуацию и знаете, что задается перед просмотром вариантов.

Правильный ответ: число \(\text{2}\).

Рабочий пример 20: третий закон Ньютона: книга и стена

Бриджит прижимает книгу к вертикальной стене, как показано на фотографии.

  1. Начертите диаграмму сил с обозначением всех сил, действующих на книгу.

  2. Изложите словами третий закон движения Ньютона.

  3. Назовите пары сил действие-противодействие, действующие в горизонтальной плоскости.

Нарисуйте диаграмму сил

Диаграмма сил будет выглядеть следующим образом:

Обратите внимание, что нам нужно было нарисовать все силы, действующие на книгу, а не пары действие-противодействие. Ни одна из нарисованных сил не является парой действие-противодействие, поскольку все они действуют на один и тот же объект (т. книга). Когда вы маркируете силы, будьте как можно более конкретными, включая направление силы. и оба задействованных объекта, например, не говорят о гравитации (что является неполным ответом), а скорее скажем «Нисходящая (направление) гравитационная сила Земли (объекта) на книгу (объект)’.

Сформулируйте третий закон Ньютона

Если тело А действует на тело В с силой, то тело В будет действовать с такой же по величине силой, но в противоположном направлении, на кузов А.

Назовите пары действие-противодействие

В вопросе задаются только силы действия-противодействия в горизонтальной плоскости. Следовательно:

Пара 1: Действие: Бриджит приложила силу к книге; Реакция: сила книги на девочка.

Пара 2: Действие: Сила книги на стене; Реакция: Сила стены на книге.

Обратите внимание, что пара третьего закона Ньютона всегда будет включать одну и ту же комбинацию слов, например «книга». на стене» и «стена на книге». Объекты «меняются местами» при именовании пар.

временный текст

Воздушный шар ракеты

Цель

В этом эксперименте для всего класса вы будете использовать ракету-шар, чтобы исследовать Ньютона. третий закон. Леска будет использоваться в качестве дорожки, а пластиковая соломка, прикрепленная скотчем к воздушному шару, будет помогите прикрепить воздушный шар к дорожке.

Аппарат

Для этого эксперимента вам понадобятся следующие предметы:

  1. воздушных шаров (по одному на каждую команду)

  2. пластиковые соломинки (по одной на каждую команду)

  3. лента (целлофановая или малярная)

  4. леска, \(\text{10}\) метров в длину

  5. секундомер – опционально (можно также использовать мобильный телефон)

  6. измерительная лента – опционально

Метод

  1. Разделитесь на группы не менее чем по пять человек.

  2. Прикрепите один конец лески к доске скотчем. Пусть один товарищ по команде держит другой конец лески, чтобы он был натянут и примерно горизонтален. Линия необходимо держать устойчиво и нельзя перемещать вверх или вниз во время эксперимент.

  3. Попросите одного из товарищей надуть воздушный шарик и зажать его пальцами. Иметь другой товарищ по команде прикрепляет соломинку к воздушному шару. Проденьте леску через соломинку и держите воздушный шар в дальнем конце линии.

  4. Отпустите ракету и посмотрите, как ракета движется вперед.

  5. При желании ракеты каждой группы можно засечь, чтобы определить победителя самой быстрой ракета.

    1. Назначьте одного товарища по команде для измерения времени события. Воздушный шар следует отпустить, когда время Вратарь кричит: «Вперед!» Наблюдайте, как ваша ракета движется к доска.

    2. Попросите другого члена команды встать рядом с доской и кричать “Останавливаться!” когда ракета попадает в цель. Если баллон не дойти до доски: «Стой!» следует вызывать, когда воздушный шар перестает двигаться. Хронометрист должен зафиксировать время полета.

    3. Измерьте точное расстояние, пройденное ракетой. Вычислите среднюю скорость на который путешествовал воздушный шар. Для этого разделите пройденное расстояние на время, когда воздушный шар находился «в полете». Введите свои результаты для пробной версии 1. в таблице ниже.

    4. Каждая команда должна провести еще две гонки и заполнить разделы таблицы для испытаний 2 и 3. Затем рассчитайте среднюю скорость для трех испытаний, чтобы определить время входа в гонку вашей команды.

Результаты

9{-1}$}\))

Испытание 1

Испытание 2

Испытание 3

В среднем:

Выводы

Победителем этой гонки становится команда с наибольшей средней скоростью воздушного шара.

Видео: 23JB

Во время эксперимента вы должны подумать о том,

  1. Что заставило вашу ракету двигаться?

  2. Как эта деятельность демонстрирует третий закон движения Ньютона?

  3. Нарисуйте изображения с помощью стрелок, чтобы показать силы, действующие на внутреннюю часть воздушного шара перед он был выпущен и после того, как он был выпущен.

Видео: 23JC

Видео: 23JD

Saturn V (произносится как «Сатурн Пять») — американская ракета одноразового использования, предназначенная для людей. Программы НАСА «Аполлон» и «Скайлэб» с 1967 до 1973 года. Многоступенчатая ракета-носитель на жидком топливе, НАСА. запустил 13 самолетов Saturn V из Космического центра Кеннеди, Флорида, без потери экипажа или полезной нагрузки. Это остается самая высокая, самая тяжелая и самая мощная ракета, когда-либо доведенная до рабочего состояния и до сих пор удерживающая Рекорд по самой тяжелой полезной нагрузке ракеты-носителя.

Учебник Упражнение 2.6

Муха попала в лобовое стекло движущегося автомобиля. По сравнению с величиной силы муха давит на ветровое стекло, величина силы, с которой ветровое стекло действует на муха во время столкновения, составляет:

  1. ноль.

  2. меньше, но не ноль.

  3. больше.

  4. то же.

то же самое

Какая из следующих пар сил правильно иллюстрирует третий закон Ньютона?

А или В

Равновесные силы (ESBKW)

В начале этой главы упоминалось, что равнодействующие силы заставляют объекты ускоряться в прямая линия. Если объект неподвижен или движется с постоянной скоростью, то либо

Другими словами, для неподвижных объектов или объектов, движущихся с постоянной скоростью, результирующая сила действие на объект равно нулю.

Равновесие

У объекта, находящегося в равновесии, сумма действующих на него сил равна нулю.

временный текст

Предыдущий

2.2 Сила

Оглавление

Следующий

2. 4 Силы между массами

Законы Ньютона

Законы Ньютона
Индекс
 
Гиперфизика***** Механика R Ступица
Назад

Первый закон Ньютона гласит, что объект будет оставаться в состоянии покоя или в равномерном прямолинейном движении, если на него не действует внешняя сила. Это можно рассматривать как утверждение об инерции, что объекты останутся в своем состоянии движения, если сила не изменит движение. Любое изменение в движении связано с ускорением, и тогда применяется второй закон Ньютона. Первый закон можно рассматривать как частный случай Второго закона, для которого результирующая внешняя сила равна нулю, но который содержит некоторые предположения относительно системы отсчета, в которой рассматривается движение. Утверждения как Второго закона, так и Первого закона здесь предполагают, что измерения производятся в системе отсчета, которая сама по себе не ускоряется. Такой кадр часто называют «инерциальным кадром». Формулировка этих законов должна быть обобщена, если вы имеете дело с вращающейся системой отсчета или любой системой, которая ускоряется.

Первый закон Ньютона содержит следствия относительно фундаментальной симметрии Вселенной в том смысле, что состояние движения по прямой линии должно быть таким же «естественным», как и состояние покоя. Если объект находится в состоянии покоя в одной системе отсчета, он будет казаться движущимся по прямой линии наблюдателю в системе отсчета, которая движется мимо объекта. Невозможно сказать, какая система отсчета является «особой», поэтому все системы отсчета с постоянной скоростью должны быть эквивалентны.

Индекс

Масса на примере струны.

Концепции законов Ньютона.

 
Гиперфизика***** Механика R Ступица
Назад

Струна должна создавать центростремительную силу, необходимую для движения мяча по кругу. Если струна оборвется, мяч будет двигаться по прямой линии. Прямолинейное движение в отсутствие сдерживающей силы является примером первого закона Ньютона. В приведенном здесь примере предполагается, что никакие другие результирующие силы, такие как горизонтальное движение на поверхности без трения, не действуют. Вертикальный круг более сложный.

Расчет центростремительной силы
Индекс
 
Гиперфизика***** Механика R Ступица
Назад
Index

Понятия законов Ньютона.

 
Гиперфизика***** Механика R Ступица
Назад

Второй закон Ньютона, изложенный ниже, применим к широкому кругу физических явлений, но не является фундаментальным принципом, как законы сохранения. Это применимо только в том случае, если сила является чистой внешней силой. Он не применяется непосредственно к ситуациям, когда масса изменяется либо из-за потери, либо из-за увеличения материала, или из-за того, что объект движется со скоростью, близкой к скорости света, когда необходимо учитывать релятивистские эффекты. Она неприменима непосредственно к очень маленькому размеру атома, где необходимо использовать квантовую механику.

Данные можно ввести в любое из полей ниже. Задание любых двух величин определяет третью. После того, как вы ввели значения для двух, щелкните текст, представляющий третий, чтобы вычислить его значение.


ньютонов = кг * м/с 2
фунтов = слизняки * фут/с 2
Ограничения второго закона Ньютона
Index

Понятия второго закона

Пример визуализации.

 
Гиперфизика***** Механика R Ступица
Назад
Второй закон Ньютона позволяет нам сравнивать результаты действия одной и той же силы на объекты разной массы.
Индекс
 
Гиперфизика***** Механика R Ступица
Назад

Третий закон Ньютона: Все силы во Вселенной действуют равными, но противоположно направленными парами. Нет изолированных сил; для каждой внешней силы, действующей на объект, существует сила равной величины, но противоположного направления, которая действует обратно на объект, воздействовавший на эту внешнюю силу. В случае внутренних сил силе, действующей на одну часть системы, противодействует сила реакции, действующая на другую часть системы, так что изолированная система никоим образом не может оказывать чистую силу на систему в целом. Система не может «запустить» себя в движение с помощью чисто внутренних сил — чтобы получить результирующую силу и ускорение, она должна взаимодействовать с внешним по отношению к ней объектом.

Без уточнения природы или происхождения сил, действующих на две массы, 3-й закон Ньютона гласит, что если они возникают от самих двух масс, они должны быть равны по величине, но противоположны по направлению, чтобы не возникало результирующей силы. от чисто внутренних сил.

Третий закон Ньютона — один из фундаментальных принципов симметрии Вселенной. Поскольку у нас нет примеров его нарушения в природе, это полезный инструмент для анализа ситуаций, которые несколько противоречат интуиции. Например, когда небольшой грузовик сталкивается лоб в лоб с большим грузовиком, ваша интуиция может подсказать вам, что сила, действующая на маленький грузовик, больше. Не так!

Маленький грузовик,
большой грузовик

Пример третьего закона Ньютона
Индекс

Пример

Концепции законов Ньютона.

 
Гиперфизика***** Механика R Ступица
Назад

Третий закон Ньютона можно проиллюстрировать, определив пары сил, которые участвуют в поддержании блоков на пружинных весах.

Если предположить, что блоки поддерживаются и находятся в равновесии, то результирующая сила, действующая на систему, равна нулю. Все силы возникают в парах по третьему закону Ньютона.

Индекс
 
Гиперфизика***** Механика R Ступица
Назад

Законы движения Ньютона Определение и значение

  • Основные определения
  • Викторина
  • Примеры
  • Научные
  • 9 Культурные

    4 0 0 Слово!


    pl n

    три закона механики, описывающие движение тела. Первый закон гласит, что тело остается в покое или в равномерном прямолинейном движении, если на него не действует сила. Второй закон гласит, что скорость изменения импульса тела пропорциональна вызывающей его силе. Третий закон гласит, что когда на тело действует сила, обусловленная другим телом, то на это тело одновременно действует равная и противоположно направленная сила

    ВИКТОРИНА

    Сыграем ли мы в «ДОЛЖЕН» ПРОТИВ. “ДОЛЖЕН” ВЫЗОВ?

    Должны ли вы пройти этот тест на «должен» или «должен»? Это должно оказаться быстрым вызовом!

    Вопрос 1 из 6

    Какая форма используется для указания обязательства или обязанности кого-либо?

    Слова рядом Законы движения Ньютона

    Телескоп Ньютона, Ньютон, Исаак, ньютон-метр, Колыбель Ньютона, Закон тяготения Ньютона, Законы движения Ньютона, Метод Ньютона, Кольца Ньютона, Новый город, Ньютаунэбби, Ньютаун Сент-Босуэллс

    Английский словарь Коллинза – полное и полное цифровое издание 2012 г. © William Collins Sons & Co. Ltd., 1979, 1986 © HarperCollins Publishers 1998, 2000, 2003, 2005, 2006, 2007, 2009, 2012

    Как использовать законы движения Ньютона в предложении

    • Согласно Pew, в 14 из 20 стран Ближнего Востока и Северной Африки есть богохульство законы.

      В защиту богохульства|Майкл Томаски|9 января 2015 г.|DAILY BEAST

    • Взгляните на это показательное исследование Pew о законах о богохульстве и отступничестве во всем мире.

      В защиту богохульства|Майкл Томаски|9 января 2015|DAILY BEAST

    • Интересно, что сейчас делает эта дама, и знает ли она, что затеяла с Арчером?

      Создатель сериала «Лучник» Адам Рид раскрывает секреты 6-го сезона, от сюрреалистических сюжетных линий до жизни после ИГИЛ|Марлоу Стерн|8 января 2015 г.|DAILY BEAST

    • Бен не православный и не особенно привержен соблюдению традиционных еврейских законов.

      Моя неделя в еврейском Tinder|Эмили Шайр|5 января 2015|DAILY BEAST

    • Таким образом, та же гниль верховенства закона, которую администрация навлекла на иммиграцию, теперь терзает наши законы о наркотиках.

      Политика Обамы в отношении марихуаны – это безумие|Джеймс Пулос|5 января 2015 г.|DAILY BEAST

    • В этой ситуации мы ждали движения врага, не замечая его продвижения к нам.

      Жизнь и самые удивительные приключения Робинзона Крузо, Йорка, моряка (1801) | Даниэль Дефо

    • Эти секции также имеют собственные вибрации, которые имеют более короткую длину и более быстрое движение.

      Культура выразительного голоса|Джесси Элдридж Саутвик

    • — Но законы Пололандии и Флатландии разные, — сказал Амалаток, начиная новое возражение.

      Великан Севера|Р.М. Баллантайн

    • Фелипе заботился о ней, как любовник; ее большие печальные глаза следили за каждым его движением.

      Рамона|Хелен Хант Джексон

    • Потому что вселенная управляется законами, и нет никаких заслуживающих доверия случаев приостановления действия этих законов.

      Бог и мой ближний|Роберт Блатчфорд

    Научные определения законов движения Ньютона

    Законы движения Ньютона


    Три закона, предложенные сэром Исааком Ньютоном относительно отношений между силой, движением, ускорением, массой и инерцией . Эти законы составляют основу классической механики и сыграли важную роль в закреплении понятий силы, массы и инерции. ♦ Первый закон Ньютона гласит, что покоящееся тело остается в покое, а движущееся тело остается в движении с постоянной скорость, если на нее не действует сила. Этот закон также называют законом инерции. ♦ Второй закон Ньютона гласит, что сила, действующая на тело, равна ускорению этого тела, умноженному на его массу. Математически выражаясь, F = ma, где F — сила в ньютонах, m — масса тела в килограммах, а a — ускорение в метрах в секунду за секунду. ♦ Третий закон Ньютона гласит, что для каждого действия существует равное и обратная реакция. Таким образом, если одно тело действует на другое тело с силой F, то на первое тело также действует сила той же силы, но в противоположном направлении. Этот закон лежит в основе конструкции ракетного двигателя, в котором вещество, выталкиваемое из горелки на высоких скоростях, создает равную силу, толкающую ракету вперед.

    Научный словарь American Heritage® Авторские права © 2011. Опубликовано издательством Houghton Mifflin Harcourt Publishing Company. Все права защищены.

    Культурные определения законов движения Ньютона

    Законы движения Ньютона


    Три закона, управляющие движением материальных объектов. Впервые они были записаны Исааком Ньютоном в семнадцатом веке и породили общее представление о природе, известное как Вселенная с часовым механизмом. Законы таковы: (1) Каждый объект движется по прямой линии, если на него не действует сила. (2) Ускорение объекта прямо пропорционально приложенной результирующей силе и обратно пропорционально массе объекта. (3) На каждое действие есть равное и противоположное противодействие.

    примечания к законам движения Ньютона

    До начала двадцатого века считалось, что эти три закона вместе с законами термодинамики и уравнениями Максвелла объясняют всю физическую вселенную.

    Новый словарь культурной грамотности, третье издание Авторское право © 2005 г., издательство Houghton Mifflin Harcourt Publishing Company. Опубликовано издательством Houghton Mifflin Harcourt Publishing Company. Все права защищены.

    Что такое законы Ньютона? – WorldAtlas

    Формула второго закона Ньютона.

    Знаменитых законов движения Ньютона три. Эти законы заложили основы ньютоновской механики, иначе известной как классическая механика. Ньютоновская механика — это область, которая сосредоточена на наборе законов, управляющих поведением объекта после того, как на этот объект действуют силы.

    Законы движения Ньютона

    Эти три закона были написаны в разных формах на протяжении столетий, по крайней мере три, но их можно кратко выразить следующим образом:

    Первый закон гласит, что объект либо остается неподвижным, либо продолжает двигаться с постоянной скоростью, если на него не воздействует другая сила. Этот закон предполагает, что объект находится в инерциальной системе отсчета. Инерциальная система отсчета – это система, в которой силы, действующие на тело, статическое или неподвижное, имеют результирующую силу, равную нулю. Эта система отсчета означает, что это тело будет оставаться неподвижным или продолжать двигаться с постоянной скоростью.

    Второй закон также предполагает, что объект находится в инерциальной системе отсчета. Закон гласит, что общий вектор сил (обозначается F) на тело эквивалентен произведению массы (обозначается m) этого тела на его ускорение (обозначается a). Математически это означает, что: F = m*a. Еще одно предположение, которое следует иметь в виду, состоит в том, что масса не меняется.

    Третий закон движения более известен. Когда одна сущность прикладывает силу (F) к другому объекту, то второе тело также будет отталкиваться с силой, равной F. На каждое действие есть равная и противоположная реакция.

    История и обзор

    Три закона движения были первоначально сформулированы никем иным, как Исааком Ньютоном, отсюда и название законов движения Ньютона. Ньютон впервые написал правила движения в 1687 году в своем выпуске 9.0039 Математические основы натуральной философии ( Philosophiæ Naturalis Principia Mathematica на латыни).

    Исаак Ньютон стремился объяснить, почему объекты ведут себя именно так, как они ведут себя, когда находятся в движении, или почему они остаются так, как они ведут себя, то есть в неподвижном состоянии. Следовательно, он использовал законы вместе с другими своими законами для объяснения движения систем, а также физических объектов.

    Еще одна важная особенность законов Ньютона заключается в том, что они применимы к объектам, которые считаются одноточечными массами. Этот термин означает, что форма и размер объекта игнорируются, чтобы сосредоточиться на его движении. Это представление применимо, если объекты малы по сравнению с расстояниями, которые учитываются при их анализе. Этот способ позволяет представить любой объект, независимо от его размера, как анализируемую частицу.

    Как было сказано ранее, трех законов недостаточно, чтобы объяснить поведение движения всех объектов. Например, он не мог объяснить законы движения планет Кеплера, пока не объединил свои законы движения с другим своим законом, названным законом всемирного тяготения. Эти законы также нельзя использовать для объяснения движения деформируемых и твердых тел. Фактически, именно в 1750 году Леонард Эйлер обобщил законы движения Ньютона, чтобы их можно было применять к твердым и деформируемым объектам, которые также считались континуумом. В законах Эйлера, которые могут быть получены из первоначальных законов Ньютона, предполагается, что объект представляет собой набор дискретных частиц, каждая из которых подчиняется законам Ньютона. Однако законы Эйлера можно считать аксиомами, описывающими законы движения протяженных объектов, не зависящих от строения частиц.

    Как указывалось ранее, законы Ньютона применимы только к набору систем отсчета, называемых инерциальными системами отсчета, которые иногда называют ньютоновскими системами отсчета. Однако среди ученых были некоторые споры относительно первого и второго законов. Одна школа мысли утверждает, что первый закон Ньютона описывает, что такое инерциальная система отсчета, и поэтому второй закон верен, если и только если он наблюдается с точки зрения инерциальной системы отсчета. При рассмотрении всех этих факторов невозможно определить особый из двух законов. Другая школа мысли утверждает, что первый закон является следствием второго.

    Еще один аспект этих законов, о котором следует помнить, заключается в том, что специальная теория относительности устарела по сравнению с законами Ньютона. Это не значит, что они бесполезны. Законы подходят для аппроксимации поведения движущихся объектов, когда их скорости ниже скорости света.

    Три закона в деталях

    Первый закон

    Первый закон Ньютона гласит, что скорость движущегося объекта должна оставаться постоянной, если результирующая сила равна нулю. В этом случае сила относится к векторной сумме всех сил, воздействующих на это тело. Скорость является векторной величиной, так как она показывает не только скорость тела, но и направление его движения. Это означает, что постоянная скорость описывает постоянное направление и скорость объекта.

    Если выразить это с помощью математической формулы, то получится: ∑F = 0 ↔ d v /d t = 0. В формуле v представляет собой скорость, а t представляет затраченное время. . Формула лишь доказывает, что неподвижный объект останется таким, если на него не будет воздействовать сила, а движущееся тело не изменит своей скорости, если на него не будет воздействовать сила. Такое движение называется равномерным. Хороший способ продемонстрировать это — провести эксперимент со скатертью. Блюда, поставленные поверх скатерти, останутся такими, какие они есть, если скатерть снять умело и быстро. Это не трюк, а законы Ньютона в действии. Естественная тенденция движущегося объекта — оставаться таким, какой он есть. Если кто-то хочет изменить эту тенденцию, то к этому объекту нужно применить силу. Этот закон также устанавливает системы отсчета для двух других законов.

    Второй закон

    Другой способ сформулировать второй закон — это скорость изменения импульса объекта в прямой зависимости от количества приложенной силы. Кроме того, это изменение его импульса происходит в том же направлении, что и приложенная сила.

    Математически это можно выразить как F = d p /d t = d (m v )/d t. p является произведением массы ( m ) и скорости ( v ), тогда как t представляет затраченное время. Формула является одним из способов выразить это, однако ее также можно выразить в терминах ускорения объекта. При формулировке законов предполагается, что масса постоянна. Поэтому нет необходимости включать его в формулу дифференцирования. Таким образом, получается: F = m (d v / d t ). Поскольку скорость ( v ) деленная на время ( t ) обеспечивает ускорение, формула теперь принимает вид F = m * a .

    Масса, приобретаемая или теряемая сущностью, также влияет на импульс объекта, который не является следствием внешней силы, и становится необходимым другое уравнение. Кроме того, при более высоких скоростях расчет произведения массы объекта в состоянии покоя на его скорость неточен.

    Импульс

    Импульсы ( J ) имеют место, когда сила ( F ) действует на объект в течение интервала времени (Δt), поскольку ее математическое выражение намного ближе к формулировке второго закона Ньютона. Понятие импульса в основном используется при анализе столкновений. Математически это становится: J = Δ p = m* Δ v .

    Для систем с переменной массой, скажем, для ракеты, сжигающей топливо, второй закон неприменим, потому что они открыты. Таким образом, неправильно делать его массу функцией своего времени.

    Третий закон Ньютона

    Последний закон движения гласит, что все силы, существующие между двумя телами, действуют с одинаковой величиной и в противоположных направлениях. Например, если объект 1 оказывает силу величины F₁ на другое тело 2, то третий закон Ньютона гласит, что объект 2 должен оказывать силу величины -F₁, так что F₁ = – F₁. Результирующая суммарная сила равна нулю. То есть F₁ + (- F₁) = 0.

    Этот закон показывает, что все возникающие силы являются прямым следствием взаимодействия различных тел. Это также показывает, что сила не может существовать без своего равного и противоположного эквивалента, который нейтрализует ее. Направление и величина силы могут определяться одной из сил. Например, объект 1 может быть той силой, которая оказывает воздействие, и поэтому она называется силой «действия», а сила от объекта 2 называется силой «реакции». Из-за этих двух названий третий закон иногда называют законом «действия-противодействия». Однако иногда невозможно установить, какая из двух сил является действием, а какая противодействием. Одна сила не может существовать без другой. Практический пример этого — когда кто-то идет. Они упираются в землю, а земля отталкивает.

    Морин Шисия в мире Факты

    Почему важен первый закон Ньютона?

    Исаак Ньютон – Важные фигуры в истории

    50 глупых законов со всех концов США

    10 самых противоречивых законов, когда-либо принятых

    Бывшие британские колонии

    Является ли Англия частью Европы?

    История Олимпийских игр

    Страны Юго-Восточной Азии

    Сколько стран в Океании?

    Австралия — страна или континент?

    Турция находится в Европе или Азии?

    Сколько стран признано Соединенными Штатами?

    ньютоновской механики – Как можно вывести уравнения SUVAT из законов движения Ньютона?

    спросил

    Изменено 1 год, 6 месяцев назад

    Просмотрено 249 раз

    $\begingroup$

    Я бы сказал, что уравнения SUVAT, которые правильно описывают все движений с постоянным ускорением (включая гипотетические ситуации, управляемые необычными законами динамики), никак не могут быть получены из определенного набора законов динамики, таких как законы Ньютона. движение.

    Кроме того, вывод уравнений SUVAT хорошо известен, например, в статье Как выводятся уравнения SUVAT? и Происходят ли уравнения движения SUVAT (кинематика) из какого-то дифференциального уравнения? Эти выводы не имеют ничего общего ни с какими законами динамики.

    Тем не менее, ответов с наибольшим количеством голосов на активный вопрос на тему SUVAT гласит:

    На самом деле это три результата, полученные в результате перегонки законов Ньютона: $$\mathbf f = \dfrac {\mathrm d} {\mathrm d t} (m \mathbf v)$$

    , которое решается дифференциальным уравнением, где $\mathbf f$ задано константой (и $m$ также принимается как константа).

    (этот вопрос был основан на трех уравнениях СУВАТ)

    Когда я оспорил это утверждение, используя те же аргументы, что и приведенные выше, и попросил более подробную информацию, я не получил никаких ответов, которые помогли бы мне. Я получил этот ответ:

    Что касается «бессмысленности» вывода этих уравнений движения из уравнений Ньютона, то именно так это было введено в одном из моих курсов по прикладной математике где-то во время моего обучения — как способ объединения уравнений СУВАТ с факты действительности.

    Другой пользователь, не участвовавший в этом чате, отметил вопрос как newtonian-mechanics.

    Таким образом, идея о том, что уравнения SUVAT можно вывести из законов движения Ньютона, кажется общепризнанной. Где я ошибся, утверждая, что такой вывод невозможен, и что такое вывод?

    • ньютоновская механика
    • кинематика

    $\endgroup$

    $\begingroup$

    Возможно, уравнения suvat больше математические, чем физические. Мы начнем с аксиом — определения скорости через перемещение и время и ускорения через скорость и время — и выведем уравнения. Действительно, я полагаю, что сам Галилей считал кинематику частью геометрии. И это было более чем за 250 лет до Минковского!

    Я полагаю, что учитель мог бы мотивировать изучение постоянного ускорения, рассматривая тело, на которое действует постоянная сила, например тележку, которую тянет груз, соединенный с тележкой веревкой, проходящей через шкив, но Я надеюсь, что позже учитель укажет, что сами уравнения не требуют специальной настройки. 2 = d(mv)/dt$. В этой вселенной при постоянной силе и постоянной массе у нас все еще было бы постоянное ускорение, и уравнения SUVAT по-прежнему выполнялись бы.

    И я полагаю, что это правда, но это не значит, что уравнения SUVAT нельзя вывести из второго закона Ньютона при определенных предположениях; это просто означает, что есть другие наборы предположений, которые приведут к тому же результату. Другими словами, второй закон Ньютона (с постоянной силой и массой) подразумевает уравнения SUVAT; но знание того, что уравнения SUVAT выполняются во всех случаях для постоянной силы и массы, не означает второй закон Ньютона.

    $\endgroup$ 92} = \frac {dv}{dt} = a$

    вместе с начальными условиями $x(0) = 0$ и $v(0) = u$. Здесь нет упоминания о силе, поэтому они не имеют логической связи с законами Ньютона. Если бы объект двигался с постоянным ускорением, потому что его приводили в движение розовые единороги, а не по законам Ньютона, уравнения SUVAT все равно применялись бы к нему.

    $\endgroup$

    0

    Твой ответ

    Зарегистрируйтесь или войдите в систему

    Зарегистрируйтесь с помощью Google

    Зарегистрироваться через Facebook

    Зарегистрируйтесь, используя адрес электронной почты и пароль

    Опубликовать как гость

    Электронная почта

    Требуется, но никогда не отображается

    Опубликовать как гость

    Электронная почта

    Требуется, но не отображается

    Нажимая «Опубликовать свой ответ», вы соглашаетесь с нашими условиями обслуживания, политикой конфиденциальности и политикой использования файлов cookie

    .

    Закон тяготения Ньютона – формула, вывод, примеры

    Универсальный закон всемирного тяготения Ньютона устанавливает связь между каждой частицей и любой другой частицей во Вселенной.

    Публикация теории называется « Первое Великое Объединение », так как она ознаменовала объединение явления гравитации с известным астрономическим поведением.

    История

    Закон всемирного тяготения Ньютона был сформулирован Сэр Исаак Ньютон в 1687 и использовал для объяснение движения планет и их соответствующих лун , которые позже были сведены к математической формуле Иоганном Кеплером в начале 17 века .

    Сэр Исаак Ньютон – Сформулированный закон тяготения Ньютона (Источник)

    Закон тяготения Ньютона Закон тяготения

    Закон тяготения Ньютона утверждает, что

    «Каждая частица притягивает любую другую частицу во Вселенной с силой, которая прямо пропорциональна произведению их масс и обратно пропорциональна квадрату расстояния между их центрами».

    Итак, что такое гравитация? Гравитация — это сила, которая притягивает или притягивает любые два тела друг к другу. Каждый объект в этой вселенной притягивает любой другой объект, за исключением того, что в большинстве случаев эта сила притяжения очень слаба. Это можно объяснить большим расстоянием между объектами.

    Очень распространенным примером гравитации является то, что люди могут стоять на земле, а не парить в воздухе. Земля обладает значительной гравитационной силой. Эта сила тяжести вызывает ускорение свободного падения примерно в 9{-2}\)
    \(m_1\) – Масса 1-го -го объекта , измеренная в килограммах (кг)
    \(m_2\) – Масса 2-го -го -го объекта, измеренная в килограммах (кг)
    \(r \) – Расстояние между центрами масс обоих тел, измеренное в метрах (км).

    Из двух масс более тяжелая называется исходной массой , а другая называется тестовой массой .

    Иллюстрация гравитационной силы между двумя объектами. 2} \overrightarrow{r_{21}}\) 92}(\hat{r_{21}})\)

    \(∴ \overrightarrow{F_{12}} = – \overrightarrow{F_{21}}\)

    Следовательно, приложенные силы равны и противоположны . Это показывает, что сила гравитации подчиняется третьему закону Ньютона.

    Применение закона тяготения Ньютона
    • Ньютона l aw g гравитации помогает в прогнозировании орбит и периодов времени современных искусственных спутников.
    • Этот закон определяет направленную вниз силу на поверхности земли.
    • Закон всемирного тяготения используется для расчета силы притяжения между двумя частицами или объектами.
    • Используется для расчета значения ускорения свободного падения g над поверхностью Земли.

    Решенные примеры

    Вопрос. Какая сила притяжения действует на объект массой 300 кг у поверхности Земли?

    Раствор. Учитывая данные, которые у нас есть на данный момент,

    Масса Земли (\(M\)) = 5,98 * 10 24 93 \, N
    \end{align}
    \)

    Часто задаваемые вопросы

    Что утверждает закон всемирного тяготения Ньютона?

    Закон тяготения Ньютона гласит, что каждая частица притягивает любую другую частицу во Вселенной с силой, прямо пропорциональной произведению их масс и обратно пропорциональной квадрату расстояния между их центрами.

Оставить комментарий